Sei sulla pagina 1di 62

Chapter 4

Chapter 4 Maintaining Mathematical Proficiency (p. 173)

9.

1. (5, 2) corresponds to point G.

2y + 1 x = 7x
2y + 1 x + x = 7x + x
2y + 1 = 8x

2. (2, 0) corresponds to point D.

2y + 1 1 = 8x 1

3. Point C is located in Quadrant I.

2y = 8x 1

4. Point E is located in Quadrant IV.

5.

8x 1
2
1
y = 4x
2

2y
2

xy=5
xxy=5x

1
The rewritten equation is y = 4x .
2

y = 5 x
y
1

5x
1

y = 5 + x
The rewritten equation is y = 5 + x.
6.

6x + 3y = 1
6x 6x + 3y = 1 6x
3y = 1 6x
1 6x
3
1
y = 2x
3

3y
3

1
The rewritten equation is y = 2x.
3
7.

0 = 2y 8x + 10
0 2y = 2y 2y 8x + 10

10.

y 4 = 3x + 5y
y 5y 4 = 3x + 5y 5y
4y 4 = 3x
4y 4 + 4 = 3x + 4
4y = 3x + 4
4y
4

3x + 4
4
3
y = x 1
4

3
The rewritten equation is y = x 1.
4
11. When both coordinates of a point are multiplied by a

negative number, points in Quadrant I move to QuadrantIII,


points in Quadrant II move to Quadrant IV, points in
Quadrant III move to Quadrant I, and points in QuadrantIV
move to Quadrant II.

2y = 8x + 10
2y 8x + 10
=
2
2
y = 4x 5
The rewritten equation is y = 4x 5.
8.

x + 4y 28 = 0
x + 4y 4y 28 = 0 4y
x 28 = 4y
x 28 4y
4
4
1
x + 7 = y
4

1
The rewritten equation is y = x + 7.
4

Chapter 4 Mathematical Practices (p.174)


1. Solve a simpler problem.

Suppose you work 40 hours and earn $360.


$360 $9
Hourly wage = =
40 h
1h
In the simpler problem, you earn $9 per hour. Apply the
strategy to the original problem.
$352.50 $9.4
Hourly wage = _______
=
1h
371 h
2

In the original problem, you earn $9.40 per hour.


2. Solve a simpler problem.

Suppose you drive 1250 miles and use 50 gallons of


gasoline.
1250 mi 25 mi
Gas mileage = =
50 gal
1 gal
In the simpler problem, your cars gas mileage is 25 miles
per gallon.
Apply the strategy to the original problem.
1244.5 mi 26.2 mi
Gas mileage = =
47.5 gal
1 gal
In the original problem, your cars gas mileage is 26.2 miles
per gallon.

Copyright Big Ideas Learning, LLC


All rights reserved.

Algebra 1
Worked-Out Solutions

165

Chapter 4
3. Solve a simpler problem.

y y
x2 x1

Suppose you drive 250 miles in 5 hours. At the same rate,


how long will it take you to drive 450 miles?

y = mx + b

250 mi 50 mi
Driving speed = =
5h
1h

450 h
1h
= = 9h
50
50 mi

In the simpler problem, it will take 9 hours to drive


450miles.
Apply the strategy to the original problem.
236 mi 51.3 mi
Driving speed =
4.6 h
1h
51.3 mi
Driving time = 450 mi
1h
= 450 mi

1
2

0=
2

Because the line crosses the


y-axis at (0, 2), the y-intercept
is2.

2+b
2

2 = b
1
An equation is y = x 2.
2
2. a. The y-intercept is the point where the line crosses the

y-axis, or the value of y when x = 0. For this situation,


the y-intercept is 20. In the context of this problem, the
y-intercept is the base cost of the plan, or $20 per month.
b. Sample answer: Pick two points: (0, 20) and (1500, 65)

45
y2 y1
65 20
m=
= = = 0.03
x2 x1 1500 0 1500

450 h
1h
= 8.8 h

51.3 mi
51.3

In the original problem, it will take about 8.8 hours to drive


450miles.

or

1
0 = (4) + b
2

50 mi
Driving time = 450 mi
1h
= 450 mi

1
2

1 0
24

2
1 ===
d. m =

So, for this smartphone plan, the charge is $0.03 per


megabyte of data used.
c. Sample answer:

4.1 Explorations (p.175)

y = mx + b

y2 y1 3 (1) 3 + 1 4
1. a. m =
====2
x2 x1
20
20 2
Because the line crosses the y-axis at (0, 1), the
y-intercept is 1.
An equation is y = 2x 1.
y y
x2 x1

2 2
40

4
4

2
1 = = = 1
b. m =

Because the line crosses the y-axis at (0, 2), the y-intercept
is2.
An equation is y = x + 2.

y = 0.03x + 20
So, an equation is y = 0.03x + 20.
3. Use the graph to find the slope m of the line by identifying

two points on the graph and calculating the change in y over


change in x. The y-intercept is the point (0, b) where the line
crosses the y-axis. Once you have found the values of m and
b, you can write an equation of the line by substituting them
into y = mx+ b.
4. Sample answer:
4

2
1 3 4
y2 y1
1 3
c. m =
= = = =
3
x2 x1 3 (3)
3+3
6
y = mx + b

or

2
3 = (3) + b
3
3=
2

Because the line crosses the


y-axis at (0, 1), the y-intercept
is 1.

2+b
2

1=b

2
An equation is y = x + 1.
3

166

Algebra 1
Worked-Out Solutions

(2, 0)
4

4 x

(0, 2)
4

y2 y1 2 0 2
m=
===1
x2 x1
02
2
b = 2 because the line passes through 2 on the y-axis.
An equation of the line is y = x 2.

Copyright Big Ideas Learning, LLC


All rights reserved.

Chapter 4
4.1 Monitoring Progress (pp. 177178)

4.1 Exercises (pp.179180)

1. m = 7; b = 2

Vocabulary and Core Concept Check

y = mx + b

1. A linear function that models a real-life situation is called a

y = 7x + 2

linear model.

An equation is y = 7x + 2.

2. When you are given the slope m and y-intercept b, you can

write an equation of the line by substituting them into the


slope-intercept form y = mx + b.

1
2. m = ; b = 1
3
y = mx + b

Monitoring Progress and Modeling with Mathematics

1
y = x + (1)
3
1
y = x 1
3

3. y = mx + b

y = 2x + 9
An equation is y = 2x + 9.

1
An equation is y = x 1.
3

4. y = mx + b

3. Let (x1, y1) = (0, 1) and (x2, y2) = (4, 3).

y = 0x + 5
y=5

1
y2 y1 3 1 2
m=
= = , or
x2 x1 4 0 4
2

An equation is y = 5.

Because the line crosses the y-axis at (0, 1), the y-intercept
is 1.
1
So, the equation is y = x + 1.
2
4. Let (x1, y1) = (0, 1) and (x2, y2) = (5, 3).

y = 3x
6. y = mx + b

Because the line crosses the y-axis at (0, 1), the y-intercept
is 1.
2
So, the equation is y = x 1.
5
10 + 2
40

y = 3x + 0
An equation is y = 3x.

2
y2 y1 3 (1) 3 + 1
m=
= = =
5
x2 x1
50
50

10 (2)
40

5. y = mx + b

12
4

5. m = = = , or 3

Because the line crosses the y-axis at (0, 2), the y-intercept
is2.
So, an equation is y = 3x 2.

y = 7x + 1
An equation is y = 7x + 1.
7. y = mx + b

2
y = x + (8)
3
2
y = x 8
3
2
An equation is y = x 8.
3
8. y = mx + b

6. Write g(0) = 9 as (0, 9) and g(8) = 7 as (8, 7). Find the

slope of the line through these points.


1
7 9 2
m = = , or
4
80
8
Because the line crosses the y-axis at (0, 9), the y-intercept is 9.
1
So, a function is g(x) = x + 9.
4

y2 y1 277 248 29
m=
= = = 5.8
x2 x1
50
5

= 248 +

5.8

= 248 +

5.8x

3
An equation is y = x 6.
4
9. Let (x1, y1) = (0, 2) and (x2, y2) = (3, 3).

y2 y1 3 2 1
m=
==
x2 x1 3 0 3

7. Let (x1, y1) = (0, 248) and (x2, y2) = (5, 277).

Mega watt
Initial
Rate of
hours (millions) = value + change

3
y = x + (6)
4
3
y = x 6
4

Years since
2007

Because the line crosses the y-axis at (0, 2), the y-intercept
is 2.
1
So, the equation is y = x + 2.
3

The linear model is y = 5.8x + 248.


Copyright Big Ideas Learning, LLC
All rights reserved.

Algebra 1
Worked-Out Solutions

167

Chapter 4
10. Let (x1, y1) = (0, 3) and (x2, y2) = (4, 2).

1
y2 y1 2 3 1
m=
= = =
4
x2 x1 4 0
4
Because the line crosses the y-axis at (0, 3), the y-intercept
is 3.
1
So, the equation is y = x + 3.
4
11. Let (x1, y1) = (3, 4) and (x2, y2) = (0, 0).

4
0 4 4
y2 y1
04
m=
= = = =
3
x2 x1 0 (3) 0 + 3
3
Because the line crosses the y-axis at (0, 0), the y-intercept
is 0.
4
4
So, the equation is y = x + 0, or y = x.
3
3
12. Let (x1, y1) = (0, 2) and (x2, y2) = (2, 2).

y2 y1 2 (2) 2 + 2 4
m=
= = = , or 2
x2 x1
20
20 2
Because the line crosses the y-axis at (0, 2), the y-intercept
is2.
So, the equation is y = 2x 2.

So, the equation is y = 3x + 10.


5 7 12
14. m = = , or 6
02
2
Because the line crosses the y-axis at (0, 5), the y-intercept
is 5.
So, the equation is y = 6x 5.
4 + 4
02

0
2

15. m = = , = , or 0

Because the line crosses the y-axis at (0, 4), the y-intercept
is 4.
So, the equation is y = 0x + (4), or y = 4.
24 0 24 0 24
16. m = = = , or 4
0 (6)
0+6
6
Because the line crosses the y-axis at (0, 24), the
y-intercept is 24.
So, the equation is y = 4x 24.
4
1.5

0.5 2
5 2

1
10

Because the line crosses the y-axis at (0, 3), the y-intercept
is 3.
1
So, the equation is y = x + 3.
10
19. Write f (0) = 2 as (0, 2) and f (2) = 4 as (2, 4). Find the slope

of the line through these points.


42 2
m = = , or 1
20 2
Because the line crosses the y-axis at (0, 2), the y-intercept
is 2.
y = mx + b
y = 1x + 2
y=x+2
A function is f (x) = x + 2.
20. Write f (0) = 7 as (0, 7) and f (3) = 1 as (3, 1). Find the slope

of the line through these points.


1 7 6
m = = , or 2
30
3

y = mx + b

Because the line crosses the y-axis at (0, 10), the y-intercept
is 10.

15
1.5 0

0.5
5

Because the line crosses the y-axis at (0, 7), the y-intercept
is 7.

9
10 1
13. m = = , or 3
03
3

4 (4)
02

2.53
5 0

18. m = = = =

4 2
1.5 2

8
3

17. m = = = =

Because the line crosses the y-axis at (0, 5), the y-intercept
is5.

y = 2x + 7
A function is f (x) = 2x + 7.
21. Write f (4) = 3 as (4, 3) and f (0) = 2 as (0, 2). Find

the slope of the line through these points.


1
1
2 (3) 2 + 3
m = = = =
4
04
04
4
Because the line crosses the y-axis at (0, 2), the y-intercept
is 2.
y = mx + b
1
y = x 2
4
1
A function is f (x) = x 2.
4
22. Write f (5) = 1 as (5,1) and f (0) = 5 as (0, 5). Find

the slope of the line through these points.


5 (1) 5 + 1 4 4
m====
05
05
5 5
Because the line crosses the y-axis at (0, 5), the y-intercept
is 5.
y = mx + b
4
y = x 5
5
4
A function is f (x) = x 5.
5

8
So, the equation is y = x + 5.
3

168

Algebra 1
Worked-Out Solutions

Copyright Big Ideas Learning, LLC


All rights reserved.

Chapter 4
23. Write f (2) = 6 as (2, 6) and f (0) = 4 as (0,4). Find

28. The coordinates of the points were substituted incorrectly

the slope of the line through these points.

when calculating the slope.

4 6 10
4 6
m = = = , or 5
0 (2)
0+2
2

Let (x1, y1) = (0, 4) and (x2, y2) = (5, 1).

Because the line crosses the y-axis at (0,4), the y-intercept


is 4.
y = mx + b

3
y2 y1 1 4 3
Slope =
= = =
5
x2 x1 5 0
5
Because the line crosses the y-axis at (0, 4), the y-intercept is
4, which was correct.

y = 5x 4

y = mx + b

A function is f (x) = 5x 4.

3
y = x + 4
5

24. Write f (0) = 3 as (0, 3) and f (6) = 3 as (6, 3). Find the

slope of the line through these points.


0
33
m = = , or 0
6 0 6
Because the line crosses the y-axis at (0, 3), the y-intercept
is 3.
y = mx + b
y = 0x + 3
y=3
A function is f (x) = 3.
25. Let (x1, y1) = (1,1) and (x2, y2) = (0, 1).

2
y2 y1 1 (1) 1 + 1
m=
= = = = 2
x2 x1
01
0 1 1
Because the point (0, 1) is on the y-axis, the y-intercept is 1.
y = mx + b
y = 2x + 1
A function is f (x) = 2x + 1.
26. Let (x1, y1) = (4,2) and (x2, y2) = (0, 0).

1
y2 y1 0 (2) 0 + 2 2
m=
= = = , or
x2 x1 0 (4) 0 + 4 4
2
Because the point (0, 0) is on the y-axis, the y-intercept is 0.

3
An equation is y = x + 4.
5
29. a. Let (x1, y1) = (0, 3.91) and (x2, y2) = (20, 3.81).

y2 y1 3.81 3.91 0.1 0.1 10


m=
===
x2 x1
20 0
20
20 10
1
= , or 0.005
200
World record
Initial
Rate of
(in minutes) = value + change

Years

since 1960
x
+ 0.005

= 3.91

= 3.91 0.005x

A linear model is y = 3.91 0.005x.


b. y = 3.91 0.005x

y = 3.91 0.005x

y = 3.91 0.005 (40)

y = 3.91 0.005 (60)

y = 3.91 0.2

y = 3.91 0.3

y = 3.71

y = 3.61

The model estimates the record time for the mens mile in
2000 was 3.71 minutes, and the model predicts the record
time in 2020 will be 3.61 minutes.
30. a. Words: Total

cost =

y = mx + b

Initial
Rate of
fee + Change (cost
per hour)

Studio

(in
time
hours)

1
y = x + 0
2
1
y = x
2

Variables: Let y be the total cost and x be the time


(inhours) spent in the studio.

1
A function is f (x) = x.
2

y = 50 + 75x

27. The slope and y-intercept were substituted incorrectly. The

slope is 2. So, m= 2. The y-intercept is 7. So, b = 7.


y = mx + b
y = 2x + 7
An equation is y = 2x + 7.

Copyright Big Ideas Learning, LLC


All rights reserved.

Linear model: y = 50 + 75 x
A linear model is y = 50 + 75x.
b. y = 50 + 75x

y = 50 + 75 (12)
y = 50 + 900
y = 950
According to the model, 12 hours of recording time will
cost $950. So, it would be less expensive to purchase a $750
music software program.

Algebra 1
Worked-Out Solutions

169

Chapter 4
2

31. Let (x1, y1) = (0,2) and (x2, y2) = (0, 5).

c. You can use the slope m = 9 and y-intercept b = 8

from part (a) to write a linear model that represents


this situation. Let y be the approximate U.S. box office
revenues (in billions of dollars) and x be the number of
years that have passed since the year 2000.

y2 y1 5 (2) 5 + 2 7
m=
===
x2 x1
00
00 0
no; Because you cannot divide by 0, the slope is undefined.
So, you cannot write an equation of the line in slopeintercept form.

y = mx + b
2
y = x + 8
9
Then, in order to predict the U.S. box office revenue in
2018, substitute 18 for x and solve for y.

32. Sample answer: You have $20 set aside for your savings,

and each week you plan to save another $15 out of your
babysitting earnings. Let y be how much you have saved,
andx be how long (in weeks) you have been saving.

2
y = x + 8
9
2
y = (18) + 8
9

Ax + By = C

33.

Ax + Ax + By = C Ax
By = C Ax

y=4+8

C Ax
y=
B
A
C
y = x +
B
B
In the equation 6x + 5y = 9, A = 6, B = 5, and C = 9.
9
6
So, the rewritten equation is y = x + .
5
5
9
6
So, the slope is m = , and the y-intercept is b = .
5
5

y = 12
So, based on this model, the predicted U.S. box office
revenue in 2018 is about $12 billion.
37. Let (x1, y1) = (0, b) and (x2, y2) = (1, b + m).

y2 y1 b + m b m
m=
===m
x2 x1
10
1
Because the line passes through the point (0, b), the
y-intercept is b. So, the equation is y = mx + b.

34. Your friend is correct. If f is a function, then the line is not

To be sure that the point (1, b m) lies on the line,


substitute 1 for x and b m for y.

vertical.

y = mx + b
?
b m = m(1) + b
?
b m = m + b

35. Reflect (0, 1) and (3, 4) in the x-axis. The reflected points

are (x1, y1) = (0, 1) and (x2, y2) = (3, 4).

y2 y1 4 (1) 4 + 1 5
Slope of line k =
===
x2 x1
30
30 3
Because line k passes through the point (0, 1), the
y-intercept of line k is 1.
5
So, an equation that represents line k is y = x 1.
3
36. a. Sample answer: Let (x1, y1) = (0, 8) and (x2, y2) = (9, 10).

y2 y1 10 8 2
m=
==
x2 x1
90
9
2
So, the slope is , or about 0.22.
9
Because the line appears to pass through the point (0, 8),
the y-intercept is 8.
b. Sample answer: The U.S. box office revenue was about

$8billion in 2000, and it has increased at a rate of about


$0.22billion each year from 2000 to 2012.

bm=bm
The equation b m = b m is always true. So, the point
(1, b m) is a solution of the equation y = mx + b and
lies on the line.
Maintaining Mathematical Proficiency
38.

3(x 15) =

x + 11

3(x) 3(15) = x + 11
3x 45 =
x

x + 11

2x 45 = 11
+ 45 + 45
2x = 56
2x
2

56
2

x = 28
The solution is x = 28.

170

Algebra 1
Worked-Out Solutions

Copyright Big Ideas Learning, LLC


All rights reserved.

Chapter 4
39. 4y 10 = 4(y 3)

43.

3x + 5y = 15

4y 10 = 4(y) 4(3)

Let x = 0.

4y 10 = 4y 12

3x + 5y = 15

3x + 5y = 15

3(0) + 5y = 15

3x + 5(0) = 15

10 = 8y 12

5y = 15

3x = 15

+ 12

5y
=

15
5

+ 4y

+ 4y
+ 12

2 = 8y

4
3

6 5

6d + 6 = 7 + 6d

1 2 x

(0, 3)

6d
6=7

44. x 6y = 24

5(4 3n) = 10(n 2)


5(4) 5(3n) = 10(n) 10(2)
20 + 15n = 10n 20

5
6

4x + 2y = 16

4x + 2y = 16

4(0) + 2y = 16

4x + 2(0) = 16

2y = 16

4x = 16

2y
2

4x
4

(24, 0)
16

24

32 x

(0, 4)

16
4

x = 4

The y-intercept is 8.

The x-intercept is 24.

2
3

Let y = 0.

12
10

24
6

42. 4x + 2y = 16

y = 8.

x = 24

2
1

The solution is n = 0.

16
2

x 6(0) = 24

6y = 24

The y-intercept is 4.

5n = 0
5n 0
=
5
5
n=0

0 6y = 24

y = 4

+ 20

Let x = 0.

Let y = 0.
x 6y = 24

20 + 5n = 20
+ 20

Let x = 0.
x 6y = 24

6y
6

10n 10n

The x-intercept is 4.

(0, 8)

4
2

(4, 0)
6 5

3 21

2(3d) + 2(3) = 7 + 6d

The statement 6 = 7 is false. So, there is no solution.


41.

The x-intercept is 5.

2
1

(5, 0)

2(3d + 3) = 7 + 6d

15
3

x = 5

The y-intercept is 3.

1
The solution is y = .
4

6d

3x
3

y = 3

2 8y
8
8
1
=y
4

40.

Let y = 0.

3 21

1 2 x

Copyright Big Ideas Learning, LLC


All rights reserved.

Algebra 1
Worked-Out Solutions

171

Chapter 4
b.

7x 2y = 21

45.

Let x = 0.

Let y = 0.

7x 2y = 21

7x 2y = 21

7(0) 2y = 21

7x 2(0) = 21

2y = 21

7x = 21

21
2
21
y=
2
21
The y-intercept is .
2

7x
7

2y
2

7
6
5
4
3
2
1

21
7

6 5 4 3 2 1

4 3 21

x=3
The x-intercept is 3.

Use m = 2 and the point (4, 6).


y = mx + b
6 = 2(4) + b

(0, 212)

6=
8
(3, 0)

The y-intercept is 2.

1 2 3 4 x

An equation of the line is y = 2x 2.


2.

4.2 Explorations (p.181)


1. a.
6
5
4
3

m(x x1) = y y1,

or

y y1 = m(x x1)

The point-slope form of a linear equation


is y y1 = m(x x1).
1 2 3 4 5 6 7 x

3. a. Let (x1, y1) = (4, 175) and m = 25.

2
3
4
5
6

y y1 = m(x x1)
A 175 = 25(t 4)
A = 25t 100 + 175
A = 25t + 75

1
Use m = and the point (3, 2).
2

The equation is A = 25t + 75.

y = mx + b
1
2 = (3) + b
2
3
2= +b
2
3
3

2
2
1
=b
2

yy
m = 1
x x1
(y y1)
m(x x1) =
(x x1)
(x x1)

2
1

b. Graph y = 25x + 75 using a graphing calculator.


300

( 2 32 = 42 23 = 12 )

1
The y-intercept is .
2
1
1
An equation of the line is y = x + .
2
2

172

8+b

2 = b

5 4 3

1 2 3 4 5 6 x

2
3
4

y
10
8
6
4
2

Algebra 1
Worked-Out Solutions

4. When you are given the slope and a point on a line, you can

write an equation of the line by substituting the slope for m


and the coordinates of the point for x1 and y1 in the pointslope form of an equation y y1 = m(x x1).

Copyright Big Ideas Learning, LLC


All rights reserved.

Chapter 4
5. Sample answer: To write an equation of a line that passes

through the point (1, 2) and has a slope of 3, use the


point-slope form of a linear equation, and substitute 1 for
x1, 2 for y1, and 3 for m.
y y1 = m(x x1)
y 2 = 3[x (1)]
y 2 = 3(x + 1)
An equation is y 2 = 3(x + 1).
4.2 Monitoring Progress (pp. 182184)
1.

y y1 = m(x x1)
y (1) = 2(x 3)
y + 1 = 2(x 3)
The equation is y + 1 = 2(x 3).

2. y y1 = m(x x1)

2
y 0 = (x 4)
3
2
y = (x 4)
3

2
The equation is y = (x 4).
3
10 4 6
3. m = = , or 3
31
2
y y1 = m(x x1)
y 4 = 3(x 1)
y 4 = 3(x) 3(1)
y 4 = 3x 3
+4

+4

y = 3x + 1
The equation is y = 3x + 1.
4.

1
4 (1) 4 + 1 3
m = = = , or
4
8 (4)
8+4
12
y y1 = m( x x1 )
1
y (4) = (x 8)
4
1
1
y + 4 = (x) (8)
4
4
1
y + 4 = x + 2
4
4
4
1
y = x 2
4
1
The equation is y = x 2.
4

5. Rewrite g(2) = 3 as (2, 3) and g(6) = 5 as (6, 5).

1
53 2
m = = , or
62 4
2
y y1 = m(x x1)
1
y 3 = (x 2)
2
1
1
y 3 = (x) (2)
2
2
1
y 3 = x 1
2
+3
+3
1
y = x + 2
2
1
A function is g(x) = x + 2.
2
428 302
302 176 126
3
96
63
554 428 126
= = 42
3
12 9

126
3

6. = = 42, = = 42,

C C1 = m(n n1)
C 176 = 42(n 3)
C 176 = 42(n) 42(3)
C 176 = 42n 126
+ 176

+ 176

C = 42n + 50
Because the cost increases at a constant rate, the situation
can be modeled by a linear equation. A linear model is
C = 42n + 50.
4.2 Exercises (pp. 185186)
Vocabulary and Core Concept Check
1. y 5 = 2(x + 5)

y y1 = m(x x1)
The slope of the line is m = 2, and the line passes through
the point (x1, y1) = (5, 5).
2. Let (x, y) be another point on the line, where x 3. You can

write an equation relating x and y using the slope formula


with (x1, y1) = (3, 2), (x2, y2) = (x, y), and m = 4.
y2 y1
m=
x2 x1
y (2)
4=
x3
y+2
4=
x3
(y + 2)
4(x 3) = (x 3)
(x 3)

4(x 3) = y + 2
The equation in point-slope form is y + 2 = 4(x 3).

Copyright Big Ideas Learning, LLC


All rights reserved.

Algebra 1
Worked-Out Solutions

173

Chapter 4
Monitoring Progress and Modeling with Mathematics
3. y y1 = m(x x1)

y 1 = 2(x 2)
The equation is y 1 = 2(x 2).
4. y y1 = m(x x1)

y 5 = 1(x 3)
y 5 = (x 3)
The equation is y 5 = (x 3).
5. y y1 = m(x x1)

y (4) = 6(x 7)
y + 4 = 6(x 7)
The equation is y + 4 = 6(x 7).
6.

y y1 = m(x x1)
y (2) = 5[x (8)]
y + 2 = 5(x + 8)
The equation is y + 2 = 5(x + 8).

7. y y1 = m(x x1)

y 0 = 3(x 9)
y = 3(x 9)
The equation is y = 3(x 9).
8. y y1 = m(x x1)

y 2 = 4(x 0)
y 2 = 4x
The equation is y 2 = 4x.
9. y y1 = m(x x1)

3
y 6 = [x (6)]
2
3
y 6 = (x + 6)
2
3
The equation is y 6 = (x + 6).
2
10. y y1 = m(x x1)

2
y (12) = (x 5)
5
2
y + 12 = (x 5)
5
2
The equation is y + 12 = (x 5).
5

174

Algebra 1
Worked-Out Solutions

1 (3)
31

1+3
31

4
2

11. m = = = , or 2

y y1 = m(x x1)
y 1 = 2(x 3)
y 1 = 2(x) 2(3)
y1=x6
+1

+1

y = 2x 5
The equation is y = 2x 5.
5 0
5 0
1 (4)
1+4
y y1 = m(x x1)

5
5

12. m = = = , or 1

y 0 = 1[x (4)]
y = 1(x + 4)
y = x 4
The equation is y = x 4.
42
6 (2)

42
6 + 2

2
4

1
2

13. m = = = =

y y1 = m(x x1)
1
y 2 = [x (2)]
2
1
y 2 = (x + 2)
2
1
1
y 2 = (x) (2)
2
2
1
y 2 = x 1
2
+2
+2
1
y = x + 1
2
1
The equation is y = x + 1.
2
41 3
84 4
y y1 = m(x x1)
3
y 1 = (x 4)
4
3
3
y 1 = (x) (4)
4
4
3
y 1 = x 3
4
+1
+1
3
y = x 2
4
3
The equation is y = x 2.
4

14. m = =

Copyright Big Ideas Learning, LLC


All rights reserved.

Chapter 4
9 (9) 9 + 9
3 1
3 1
y y1 = m(x x1)

10
12 2
27
5
y y1 = m(x x1)

15. m = = , or 2

y 2 = 2(x 7)

y (9) = 0(x 1)

y 2 = 2(x) 2(7)

y+9=0

y 2 = 2x + 14
+2

99

+2

y = 9

y = 2x + 16

The equation is y = 9.

The equation is y = 2x + 16.


1+2
12 6

1 (2)
12 6

3
6

1
2

16. m = = = =

y y1 = m(x x1)
1
y (2) = (x 6)
2
1
1
y + 2 = (x) (6)
2
2
1
y + 2 = x 3
2
2
2
1
y = x 5
2
1
The equation is y = x 5.
2
7 (1) 7 + 1 6
17. m = = = , or 2
36
36
3
y y1 = m(x x1)
y (1) = 2(x 6)
y + 1 = 2(x 6)
y + 1 = 2(x) 2(6)
y + 1 = 2x 12
1

y = 2x 13
The equation is y = 2x 13.
5 5 10
5 5
18. m = = = , or 5
4 (2) 4 + 2
2
y y1 = m(x x1)
y 5 = 5[x (2)]
y 5 = 5(x + 2)
y 5 = 5(x) + 5(2)
y 5 = 5x + 10
+5

0
4

19. m = = = , or 0

+5

y = 5x + 15
The equation is y = 5x + 15.

Copyright Big Ideas Learning, LLC


All rights reserved.

13 19
13 19
5 (5)
5+5
y y1 = m(x x1)
3
y 13 = (x 5)
5
3
3
y 13 = (x) (5)
5
5
3
y 13 = x + 3
5
+ 13
+ 13

6
10

3
5

20. m = = = , or

3
y = x + 16
5
3
The equation is y = x + 16.
5
21. Rewrite f (2) = 2 as (2, 2) and f (1) = 1 as (1, 1).

3
1 (2) 1 + 2
m = = = , or 3
12
1 2 1
y y1 = m(x x1)
y 1 = 3(x 1)
y 1 = 3(x) 3(1)
y 1 = 3x + 3
+1

+1

y = 3x + 4
A function is f (x) = 3x + 4.
22. Rewrite f (5) = 7 as (5, 7) and f (2) = 0 as (2, 0).

07
7
m = = , or 1
2 5 7
y y1 = m(x x1)
y 7 = 1(x 5)
y 7 = 1(x) 1(5)
y7=x5
+7

+7

y=x+2
A function is f (x) = x + 2.

Algebra 1
Worked-Out Solutions

175

Chapter 4
23. Rewrite f (4) = 2 as (4, 2) and f (6) = 3 as (6, 3).

1
3 2 5
3 2
m = = = , or
2
6 (4)
6+4
10

26. Rewrite f (9) = 10 as (9, 10) and f (1) = 2

as (1, 2).

3
2 10 12
2 10
m = = = , or
2
1 (9)
1 + 9
8

y y1 = m(x x1)

y y1 = m(x x1)
3
y (2) = [x (1)]
2
3
y + 2 = (x + 1)
2
3
3
y + 2 = (x) (1)
2
2
3
3
y + 2 = (x)
2
2
2
2

1
y (3) = (x 6)
2
1
1
y + 3 = (x) (6)
2
2
1
y + 3 = x + 3
2
3
3
1
y = x
2
1
A function is f (x) = x.
2

3 4
7
3
2 = =
2
2 2
2

3
7
A function is f (x) = x .
2
2

44
0
44
m = = = , or 0
2 (10) 2 + 10 8
y y1 = m(x x1)

15 5 10
5 (1) 5 + 1 6
42 2
64
2
42
29 15 14
==7
2
86
47 29 18
==9
2
10 8
Because the y-values are not changing at a constant rate, the
data cannot be modeled by a linear equation.

27. = = = 3, = = 5,

y 4 = 0[x (2)]
y4=0
+4

3
7
y = x
2
2

24. Rewrite f (10) = 4 as (10, 4) and f (2) = 4 as (2, 4).

+4

y=4
A function is f (x) = 4.
25. Rewrite f (3) = 1 as (3, 1) and f (13) = 5 as (13, 5).

10 16 6
10 16
2
1 (3) 1 + 3
4 10 6
4 10
= = = 3,
1 (1)
1+1
2
8 (2) 8 + 2 6
2 4 6
= = 3, = = = 3
2
53
53
2
31

51
4
1
51
m = = = , or
13 (3) 13 + 3 16
4

28. = = = 3,

y y1 = m(x x1)
1
y 5 = (x 13)
4
1
1
y 5 = (x) (13)
4
4
13
1
y 5 = x
4
4
+5

y 4 = 3(x 1)
y 4 = 3(x) 3(1)

+5

13 20 7
13
+ 5 = + =
4
4
4
4
7
1
A function is f (x) = x + .
4
4
7
1
y = x +
4
4

176

y y1 = m(x x1)

Algebra 1
Worked-Out Solutions

y 4 = 3x + 3
+4

+4

y = 3x + 7
Because the y-values decrease at a constant rate, the data can
be modeled by a linear equation. A linear model is
y = 3x + 7.

Copyright Big Ideas Learning, LLC


All rights reserved.

Chapter 4
1.6 1.4
1.4 1.2 0.2
1
21
10
0.4
2 1.6
= = 0.2
42
2

0.2
1

29. = = 0.2, = = 0.2,

y y1 = m(x x1)
y 2 = 0.2(x 4)
y 2 = 0.2(x) 0.2(4)
y 2 = 0.2x 0.8
+2

+2

y = 0.2x + 1.2
Because the y-values increase at a constant rate, the data
canbe modeled by a linear equation. A linear model is
y = 0.2x + 1.2.
15 18 3
12 15 3 9 12 3
30. = = 3, = , =
21
1
42
2 84
4
The y-values are decreasing at a constant rate, but the
x-values are not increasing at a constant rate. The rate of
change is not constant. So, the data cannot be modeled by a
linear equation.
31. In point-slope form, the slope is multiplied by the quantity

x x1. So, the equation is y y1 = m(x x1).


6
10 4
m = = = 3
35
2
y y1 = m(x x1)
y 4 = 3(x 5)
y 4 = 3(x) 3(5)
y 4 = 3x + 15
+4

+4

y = 3x + 19
A linear function is f (x) = 3x + 19.
32. The values that are substituted for x1 and y1 should be from

the same point.


32 1
m==
41 3
Let (x1, y1) = (1, 2).
y y1 = m(x x1)
1
y 2 = (x 1)
3
1
An equation is y 2 = (x 1).
3

33. a.

Words:

Total
cost (in
dollars)

Initial
Change
cost
per
= (for first + additional
1000)
1000

Number of
additional
stickers
ordered

Variables: Let y be the total cost (in dollars) and x be the


number (in thousands) of stickers ordered.

Equation: y = 225 + 80 (x 1)
y = 225 + 80x 80
y = 80x + 145
A linear equation that models this situation is
y = 80x + 145.
b. y = 80x + 145

= 80(9) + 145
= 720 + 145
= 865
The total cost of 9000 stickers is $865.
654 450
450 246 204
2
64
42
858 654 204
= = 102
86
2

204
2

34. a. = = 102, = = 102,

Because the rate of change is constant, the situation can


be modeled by a linear equation.
b. Let C be the total cost of renting the beach house and d be

the number of days of the rental.


C C1 = m(d d1)
C 246 = 102(d 2)
C 246 = 102(d) 102(2)
C 246 = 102d 204
+ 246

+ 246

C = 102d + 42
Because the situation can be modeled by the linear
equation C = 102d + 42, you know that the daily fee is
the rate of change, or m = $102, and the processing fee is
the initial charge, or b = $42.
c. 102d + 42 1200

42

42

102d 1158
102d
102

1158
102

d about 11.4
Because you cannot rent the beach house for part of a day,
the maximum number of days you can rent the beach house
is 11 days.

Copyright Big Ideas Learning, LLC


All rights reserved.

Algebra 1
Worked-Out Solutions

177

Chapter 4
35. Sample answer: Because the equation is in point-slope form

y y1 = m (x x1), we can see immediately that the line


has a slope of m = 32 and passes through the point (x1, y1) =
(4, 1). So, one way to graph the equation is to plot the point
(4, 1) and then use the slope to plot a second point. Draw a
line through these two points.
A second way to graph this equation is to first rewrite the
equation in slope-intercept form.

b. Sample answer: The coordinates of the two points appear

( )

to be about 4, 53 and (8, 4).

y = mx + b
or
7
4 = (8) + b
12
14
4=+b
3
14
14
___

___
3
3
_____
= _____
2
b
__

3
2
14 12 14
4 = =
3
3
3
3

The y-intercept is 5, so plot the point (0, 5). Then use


the slope to plot a second point. Draw a line through these
two points.
36. Sample answer:

2
Let m = and (x1, y1) = (12, 5).
5
y y1 = m (x x1)
2
y (5) = (x 12)
5
2
2
y + 5 = (x) (12)
5
5
24
2
y + 5 = x
5
5
5
5

function appears to be negative, because if you connect


the points with a straightedge, the line crosses the y-axis
below the origin.

You can use these coordinates to calculate the slope of


the line. Then, you can either use slope-intercept form to
calculate the value of b, or you can write an equation of
the line in point-slope form and rewrite the equation in
slope-intercept form to identify the value of b.
7
5
4 __
3 3
7
m===
8 4 4 12

3
y 1 = (x 4)
2
3
3
y 1 = (x) (4)
2
2
3
y 1 = x 6
2
+1
+1
3
y = x 5
2

24 25
49
24
49
2
y = x
5 = =
5
5
5
5
5
5
Two ways to represent this function are
2
49
2
and
f (x) = x .
y + 5 = (x 12)
5
5
5

38. a. Sample answer: The y-intercept of the graph of the linear

y y1 = m(x x1)
7
y 4 = (x 8)
12
7
7
y 4 = (x) (8)
12
12
14
7
y 4 = x
12
3
+4

+4

2
7
y = x
12
3
2
So, b = , which confirms that the y-intercept of the
3
graph is negative.
39. Sample answer: The graph of y 1 = 2(x + 3) is the graph

of y = 2x translated 1 unit up and 3 units left. So, the graph


of y k = m(x h) is a translation h units to the right and
k units upward of the graph of y = mx.

37. Sample answer: In order to write an equation of a the line

that passes through two points that are not on the y-axis, use
the point-slope form to write the equation because you can
quickly use the coordinates of the two points to calculate the
slope and use one of the points to write the equation in pointslope form. In order to use slope-intercept form, you would
have to calculate both the slope and the y-intercept.

178

Algebra 1
Worked-Out Solutions

Copyright Big Ideas Learning, LLC


All rights reserved.

Chapter 4
40. a. Because the graph of Sibling As spending crosses the

y-axis at (0, 8), Sibling As initial value is $80.


If you insert a row at the beginning of Sibling Bs table
by subtracting 1 in the x column and adding $25 in the
ycolumn, you get (0, 125). So, Sibling Bs initial value
is$125.
Because Sibling Cs equation, y = 22.5x + 90, is in
slope-intercept form y = mx + b, you know that b = 90,
or Sibling Cs initial value is $90.
So, Sibling B had the most money initially, and therefore
received the most money for the holiday. Sibling A had
the least amount of money initially and therefore received
the least.
b. Calculate the slope of the line that represents Sibling As

Maintaining Mathematical Proficiency


1
41. The reciprocal of 5 is .
5
1
8

42. The reciprocal of 8 is .

4.3 Explorations (p.187)


3x + 4y = 6

1. a.

3x 3x + 4y = 6 3x
4y = 6 3x

20 50 30
m = = , or 15
42
2

6 3x
4
3 3
y = x,
2 4

4y
4

The slope is m = 15. So, Sibling A spends $15 per week.


Calculate the rate of change in the table that represents
Sibling Bs spending.

4y = 12 3x
12 3x
4
3
y = 3 x, or
4

4y
4

4x + 3y = 12
3y = 12 4x

5 and 6, Sibling A runs out of money after more than


5weeks.

y = 22.5x + 90
0 = 22.5x + 90
90

90

90 = 22.5x
22.5x
90
=
22.5
22.5
4=x

3
y = x + 3
4

4x 4x + 3y = 12 4x

c. Sibling A: Because the line crosses the x-axis between

Sibling C: Let y = 0.

3
3
y = x +
4
2

3x 3x + 4y = 12 3x

Because Sibling Cs equation, y = 22.5x + 90, is


inslope-intercept form y = mx + b, you know that
m = 22.5. So, Sibling C spends $22.50 per week.

Sibling B: If you add one more row at the end of the table
by adding 1 in the x column and subtracting $25 in the
ycolumn, you get (5, 0). So, Sibling B runs out of money
after exactly 5 weeks.

or

3x + 4y = 12

The rate of change is m = 25. So, Sibling B spends


$25per week.

So, Sibling B spends money at the fastest rate, and


SiblingA spends money at the slowest rate.

2
3

3
2

44. The reciprocal of is .

spending.

75 100 25
m = = , or 25
21
1

7
2

2
7

43. The reciprocal of is .

3y 12 4x
=
3
3
4
y = 4 x, or
3

4
y = x + 4
3

y=

3
4x

y = 4x + 3

3
2

y = 3x + 4
3

The first two lines, y = 4 x + 32 and y = 4 x + 3, are


parallel because they are always the same distance apart.
Note that they have the same slope.

So, Sibling C runs out of money after exactly 4 weeks.


So, Sibling C runs out of money first, and Sibling A runs
out of money last.

Copyright Big Ideas Learning, LLC


All rights reserved.

Algebra 1
Worked-Out Solutions

179

Chapter 4
3

The first line y = 4x + 32 and the third line y = 43x 4


are perpendicular because they intersect at a right angle.
3
Note that their slopes, m = 4 and m = 43 are negative
reciprocals.

5x + 2y = 6

b.

5x 5x + 2y = 6 5x
2y = 6 5x
6 5x
2
5
y = 3 x,
2
2x + y = 3
2y
2

2x + 5y = 10

b.

or

5
y = x + 3
2

or

y = 2x + 3

2x 2x + 5y = 10 2x
5y = 10 2x
5y 10 2x
=
5
5
2
y = 2 x, or
5

2x 2x + y = 3 2x
y = 3 2x,
2.5x + y = 5

2x + y = 3

2.5x 2.5x + y = 5 2.5x


y = 5 2.5x,

or

y = 2.5x + 5

2x + 2x + y = 3 + 2x
y = 3 + 2x,

y=

5
2x

+3

y = 5 2.5x
y 5 2.5x
=
1
1

y = 5 + 2.5x,

The first line y = 2 x + 3 and the third line y = 2.5x + 5


are parallel because they are always the same distance apart.
Note that they have the same slopes because
5
m = 2 = 2.5.

4y
4

3
3
y = x +
4
2

or

4y = 12 3x
4y 12 3x
=
4
4
3
y = 3 + x,
4
4x 3y = 12

y = 2.5x 5

The first line y = 5x + 2 and the third line y = 2.5x 5


are perpendicular because they intersect at a right angle.
Note that the slope of the third line is 2.5 = 52 . So, the
slopes of the perpendicular lines are negative reciprocals.
3. Parallel lines are in the same plane and are always the same

4. The parallel lines in Exploration 1 have the same slope.

This will always be true because lines that are increasing or


decreasing at the same rate will never intersect.
or

3
y = x 3
4

5. The perpendicular lines in Exploration 2 have slopes that

are reciprocals of each other and have opposite signs. When


twolines have slopes that are negative reciprocals, the lines
will always be perpendicular.

4x 4x 3y = 12 4x
3y = 12 4x
3y 12 4x
=
3
3
4
y = 4 + x,
3
3

distance apart. Perpendicular lines are in the same plane and


intersect at right angles.

3x 3x 4y = 12 3x

4.3 Monitoring Progress (pp. 188190)

y = 3x 4

1 3 4
1 3
6 (5) 6 + 5 1
7 (2) 7 + 2 5
Line b: m = = = = 5
23
23
1

Lines a and b do not have the same slope. So, they are not
parallel.

or
4

3
2

y = 5 x + 2

4y = 6 3x

y = 4 x +

y = 2.5x 5

3x 3x + 4y = 6 3x

or

y = 2x + 3

3x + 4y = 6

6 3x
4
3 3
y = x,
2 4
3x 4y = 12

y = 2x + 3

2.5x 2.5x y = 5 2.5x

y = 2x + 3

2. a.

or

2.5x y = 5

y = 2.5x + 5

2
y = x + 2
5

4
y = x 4
3

1. Line a: m = = = = 4

y = 4x 3
6

180

Algebra 1
Worked-Out Solutions

Copyright Big Ideas Learning, LLC


All rights reserved.

Chapter 4
2.

13

y = mx + b
1
2 = (4) + b
4
2 = 1 + b
+1

y y1 = m(x x1)

+1

3=b
Using m = 14 and b = 3, an equation of the parallel line is
y= 14x + 3.
3. Line a: 2x + 6y = 3

4.3 Exercises (pp.191192)


Vocabulary and Core Concept Check
1
1
y = x
3
2

or

1. Nonvertical parallel lines have the same slope.


7

2. The slope of the other line is 5 . The slopes, 7 and 5 ,

are negative reciprocals of each other, so the lines are


perpendicular.

Line c: 6y + 18x = 9
6y + 18x 18x = 9 18x

Monitoring Progress and Modeling with Mathematics

6y = 9 18x

31 2
31
0 (3) 0 + 3 3
20 2
Line b: m = =
30 3
0 (3) 0 + 3 3
Line c: m = = =
3 (2) 3 + 2 5

3. Line a: m = = =

6y
6

or

3
y = 3x
2

Lines b and c have slopes of 3. So, they are parallel. Line a


1
has a slope of 3, which is the negative reciprocal of 3. So,
it is perpendicular to lines b and c.
4. y y1 = m(x x1)

1
y 5 = [x (3)]
3
1
y 5 = (x + 3)
3
1
1
y 5 = (x) + (3)
3
3
1
y 5 = x + 1
3
+5

2
y = x + 9
3
2

6y = 3 2x

9 18x
6
3
y = + 3x,
2

2
y 3 = (x 9)
3
2
y 3 = x + 6
3
+3
+3

An equation that represents the path of the boat is y = 3x + 9.

2x 2x + 6y = 3 2x
6y 3 2x
=
6
6
1 1
y = x,
2 3
Line b: y = 3x 8

5. The slope of the shoreline is m =


= 3.
41

+5

1
y = x + 6
3
An equation of the perpendicular line is y = 13x + 6.

Lines a and b have the same slope. So, they are parallel.
5
05
2
20
0 4 4
Line b: m = = , or 2
53
2
2
46
, or 2
Line c: m = =
54
1

4. Line a: m = =

Because they have the same slope, lines b and c are parallel.
0 (2) 0 + 2 2
1 (1) 1 + 1 2
2 2 4
Line b: m = = , or 2
24
2
1
12
Line c: m = = , or 1
1 0 1

5. Line a: m = = = , or 1

Because they have the same slope, lines a and c are parallel.
93
93 6
1 (1) 1 + 1 2
14 12 2
14 12
Line b: m = = = , or 2
1 (2) 1 + 2 1
10 8 2
Line c: m = =
63
3

6. Line a: m = = = , or 3

Because none of the lines have the same slope, none


are parallel.

Copyright Big Ideas Learning, LLC


All rights reserved.

Algebra 1
Worked-Out Solutions

181

Chapter 4
9.

7. Line a: 4y + x = 8

4y + x x = 8 x
4y = 8 x
4y 8 x
=
4
4
1
y = 2 x,
4
1
Line a has a slope of .
4
Line b: 2y + x = 8

3 = 2(1) + b
3 = 2 + b
+2
or

+2

5=b

1
y = x + 2
4

Using m = 2 and b = 5, an equation of the parallel line is


y= 2x + 5.
10. y y1 = m(x x1)

y 2 = 5(x 1)

2y + x x = 8 x

y 2 = 5(x) 5(1)

2y = 8 x
8x
2
1
y = 4 x,
2
1
Line b has a slope of .
2
Line c: 2y = 3x + 6

y = mx + b

y 2 = 5x + 5

2y
2

+2
or

+2

y = 5x + 7

1
y = x + 4
2

An equation of the parallel line is y = 5x + 7.


11.

3y x = 12
3y x + x = 12 + x

2y 3x + 6
=
2
2
3
y = x + 3
2

3y = 12 + x
12 + x
3
1
1
y = 4 + x, or y = x 4
3
3

3y
3

3
Line c has a slope of .
2
Because none of the lines have the same slope, none are
parallel.

y = mx + b
1
2 = (18) + b
3

8. Line a: 3y x = 6

2=

3y x + x = 6 + x

3y = 6 + x
6+x
3
1
y = 2 + x,
3
1
The slope of line a is .
3

6+b
6

4 = b

3y
3

Using m = 13 and b = 4, an equation of the parallel line


or

1
y = x + 2
3

Line b: 3y = x + 18
x + 18
3
1
y = x + 6
3

3y
3

1
The slope of line b is .
3
Line c: 3y 2x = 9
3y 2x + 2x = 9 + 2x
3y = 9 + 2x
3y 9 + 2x
=
3
3
2
2
y = 3 + x, or y = x + 3
3
3
2
The slope of line c is .
3

is y= 13x 4.
12.

2y = 3x + 10
3x + 10
2
3
y = x + 5
2

2y
2

y y1 = m(x x1)
3
y (5) = (x 2)
2
3
3
y + 5 = (x) (2)
2
2
3
y + 5 = x 3
2
5

3
y = x 8
2
An equation of the parallel line is y = 32x 8.

Because lines a and b have the same slope, they are parallel.

182

Algebra 1
Worked-Out Solutions

Copyright Big Ideas Learning, LLC


All rights reserved.

Chapter 4
3
4 (1) 4 + 1 3
5 (3) 5 + 3 2
2
1
6 (4) 6 + 4 2
Line b: m = = = , or
2
2 (6) 2 + 6
4
1 (6) 1 + 6 5
Line c: m = = =
0 (3)
0+3
3

13. Line a: m = = = , or

17. Line a: 4x 3y = 2

4x 4x 3y = 2 4x
3y = 2 4x

Because none of the slopes are the same or negative


reciprocals of each other, none of the lines are parallel or
perpendicular.
01
01
2 (1) 2 + 1
4 5 1
Line b: m = =
30
3
50 5
Line c: m = =
20 2

Line b:

1
3

14. Line a: m = = =

Line c: 4y + 3x = 4
4y + 3x 3x = 4 3x
4y = 4 3x
4 3x
4
3
3
y = 1 x, or y = x + 1
4
4

4y
4

Lines a and b have the same slope. So, they are parallel.
None of the slopes are negative reciprocals of each other. So,
none of the lines are perpendicular.
31
31
0 (2) 0 + 2
41 3
Line b: m = =
64 2
1 3 2
Line c: m = =
41
3

2 4x
3
2 4
4
2
y = + x, or y = x
3 3
3
3
4
y = x + 2
3

3y
3

Lines a and b have slopes of 43 . So, they are parallel. Line c


3
has a slope of 4 , the negative reciprocal of 43 . So, line c is
perpendicular to lines a and b.

2
2

15. Line a: m = = = , or 1

Line b:

6y = x
x
6
1
y = x
6

6y
6

None of the lines have the same slope. So, none are parallel.
2
Line bs slope of 32 and line cs slope of 3 are negative
reciprocals. So, lines b and c are perpendicular.

Line c: y + 6x = 1
y + 6x 6x = 1 6x

13 10 3
42
2
12 9 3
Line b: m = =
64
2
9 10 1
Line c: m = =
42
2

16. Line a: m = =

Lines a and b have slopes of 32. So, they are parallel. None
of the lines have slopes that are negative reciprocals of each
other. So, none are perpendicular.

y = 6x 2

18. Line a:

y = 1 6x, or y = 6x + 1
1

Line as slope of 6 and line bs slope of 6 are


negative reciprocals of each other. So, lines a and b are
perpendicular.None of the lines have the same slope. So,
none are parallel.
19. y y1 = m(x x1)

y 10 = 2(x 7)
y 10 = 2(x) 2(7)
y 10 = 2x + 14
+ 10

+ 10

y = 2x + 24
An equation of the perpendicular line is y = 2x + 24.
20.

y = mx + b
3
1 = (4) + b
4
1 = 3 + b
33
4 = b
3

Using m = 4 and b = 4, an equation of the perpendicular


3

line is y = 4 x 4.
Copyright Big Ideas Learning, LLC
All rights reserved.

Algebra 1
Worked-Out Solutions

183

Chapter 4
21.

2y = 8x 6

b.

8x 6
2

2y
2

1
3 = (4) + b
4

y = 4x 3

3=1+b

y y1 = m(x x1)

11
2=b

1
y 3 = [x (3)]
4
1
y 3 = (x + 3)
4
1
1
y 3 = (x) (3)
4
4
1
3
y 3 = x
4
4
+3

Using m = 14 and b = 2, an equation of the perpendicular


line is y = 14 x + 2.
1 (4)
21

a.

y = mx + b
2 = 9 + b
99
11 = b
Using m = 3 and b = 11, an equation of the parallel
line is y = 3x 11.

2y + 4x = 12
2y + 4x 4x = 12 4x
2y = 12 4x
2y 12 4x
=
2
2
y = 6 2x, or y = 2x + 6
y = mx + b

b.

y y1 = m(x x1)
1
y (2) = (x 3)
3
1
1
y + 2 = (x) (3)
3
3
1
y + 2 = x + 1
3

1
1 = (8) + b
2

3 = b
Using m = 12 and b = 3, an equation of the perpendicular
line is y = 12x 3.
4
1

23. m = = , or 4
a. y y1 = m(x x1)

y 3 = 4(x 4)
y 3 = 4(x) 4(4)
y 3 = 4x + 16
+3

y = 4x + 19
An equation of the parallel line is y = 4x + 19.

184

Algebra 1
Worked-Out Solutions

1
y = x 1
3

1=4+b
44

+3

3
1

2 = 3(3) + b

3 12 9
3
+ 3 = + =
4
4
4
4
1
9
An equation of the perpendicular line is y = x + .
4
4

26
21

1 + 4
21

24. m = = = , or 3

+3

1
9
y = x +
4
4

22.

y = mx + b

An equation of the perpendicular line is y = 3 x 1.


25. Parallel lines have the same slope, not negative reciprocal

slopes.
y y1 = m(x x1)
1
y 3 = (x 1)
4
1
1
y 3 = (x) (1)
4
4
1
1
y 3 = x
4
4
+3

+3

1 12 11
1
+ 3 = + =
4
4
4
4
11
1
An equation of the parallel line is y = x + .
4
4
1
11
y = x +
4
4

Copyright Big Ideas Learning, LLC


All rights reserved.

Chapter 4
26. The reciprocal of the slope was used, but the slopes of

29.

perpendicular lines are negative reciprocals of each other.


y y1 = m(x x1)

y + 5 = 3x + 12
5

An equation of the perpendicular line is y = 3x + 7.

A
0

y y1 = m(x x1)
3
y 0 = (x 2)
4
3
3
y = (x) (2)
4
4
3
3
y = x +
4
2

and
the sides are parallel. Similarly, opposite sides, BC

AD , are parallel because they have the same slope. So,


quadrilateral ABCD is a parallelogram.

b. no; The slopes of adjacent sides AB and BC , 2 and 3, are

not negative reciprocals. So, the sides are not perpendicular.


None of the other pairs of adjacent sides are perpendicular
either. So, quadrilateral ABCD is not a rectangle.

y y1 = m(x x1)
4
y 5 = (x 4)
3
4
4
y 5 = (x) (4)
3
3
16
4
y 5 = x
3
3

30.

6y = 2x + 4

2y = ax + 5

2x + 4
6
1
2
y = x +
3
3
1 a
=
3 2
1
a
= 2
3
2
2
= a
3

ax + 5
2
a
5
y = x +
2
2
a
3=
2
a
2 3=2
2
2y
2

6y
6

+5
1
16 15
16
+ 5 = + =
3
3
3
3

4
40
m==
11 8 3

a. yes; Opposite sides, AB and CD , have the same slope. So,

3
3
An equation that represents the new pipe is y = x + .
4
2

1
4
y = x
3
3

1
42 2
=
= , or
slope of AB
62 4
2
10 4 6

slope of BC = = , or 3
86
2
1
10 8 2

slope of CD = = , or
84
4
2
82 6

slope of AD = = , or 3
42 2

3 (1) 3 + 1 4
m===
0 (3) 0 + 3 3

+5

y = 3x + 7

28.

y + 5 = 3(x) 3(4)

27.

y (5) = 3(x 4)

y
10

An equation that represents the new bike path is y =

2
4
3 x

1
3 .

6=a

If a = 3 , then the lines will be parallel, because their slopes


1
will both be 3. If a = 6, then the lines will have slopes of
1
3 and 3, which are negative reciprocals of each other. So,
the lines are perpendicular.
80 8
80
0 (4) 0 + 4 4
8 16 8
8 16
m = = = , or 2
0 (4)
0+4
4

31. m = = = , or 2

no; The slopes of the two segments of the pucks path are
2 and 2, which are opposites but not reciprocals of each
other. So, they are not perpendicular.

Copyright Big Ideas Learning, LLC


All rights reserved.

Algebra 1
Worked-Out Solutions

185

Chapter 4
32. a. yes; The graph of the total amount paid by StudentB

3. Sample answer: The diagram was used to identify two points

crosses the y-axis at a point that is greater than the


y-intercept of Student As graph. So, StudentB had a
greater initial cost, or registration fee.

on each segment of the pucks path. Then find the slope of


each segment and compare the slopes to determine whether
they are negative reciprocals. Because they are not, you know
that your friend is incorrect about them being perpendicular.

b. no; The lines are parallel. They have the same slope and

are increasing at the same rate. So, they pay the same
monthly fee.
33. never; The slopes of perpendicular lines must be negative

reciprocals of each other. So, if one is positive, the other


must be negative.
34. always; All vertical lines are parallel to each other, and because

the y-axis is a vertical line, it is parallel to all vertical lines.


35. sometimes; If two lines have slopes that are negative

reciprocals of each other, then they are perpendicular. If they


also have the same y-intercept, then they intersect on the
y-axis to form right angles.

3 (2)
10

3+2
10

5
1

1. m = = = , or 5

Because the line crosses the y-axis at (0, 2), the y-intercept
is 2. So, the equation is y = 5x 2.
45
30

1
3

2. m = =

Because the line crosses the y-axis at (0, 5), the y-intercept is
1
5. So, the equation is y = 3x + 5.
04
0 (2)

04
0+2

4
2

3. m = = = , or 2

36. Sample answer:


y
8

Because the line crosses the y-axis at (0, 0), the y-intercept is
0. So, the equation is y = 2x + 0, or y = 2x.

Math

1 5
1 (2)

1 5
1+2

y y1 = m(x x1)
0

6
3

4. m = = = , or 2

Club

2
0

4.14.3 Quiz (p.194)

8 x

The equations of the lines that form the long sides of the
equal sign are y = 3, y = 4.5, y = 4.75, and y = 6. They
all have a slope of 0. So, they are all parallel. The lines
that form the long sides of the multiplication sign are
y= x + 1, y = x 1, y = x + 8, and y = x + 10. The
slopes of these lines are either 1 or 1, which are negative
reciprocals of each other. These four lines form four pairs of
perpendicular lines, such as y = x + 1 and y = x + 8, and
two pairs of parallel lines, such as y = x + 1 and y = x 1.
Maintaining Mathematical Proficiency
37. yes; Each x-value is paired with exactly one y-value. So, the

relation is a function.
38. no; The same x-value, 1, is paired with two different y-values,

4 and 6, and the same x-value, 1, is paired with multiple


y-values, 2, 5, and 6. So, the relation is not a function.
4.14.3 What Did You Learn? (p. 193)
1. Sample answer: The graph shows the increase in U.S. box

office revenue over time.

or

y (1) = 2(x 1)

y y1 = m(x x1)
y 5 = 2[x (2)]

y + 1 = 2(x 1)

y 5 = 2(x + 2)

So, an equation is y + 1 = 2(x 1) or


y 5 = 2(x + 2).
1 (2)
2 (3)

1 + 2
2+3

1
5

5. m = = =

y y1 = m(x x1)

or

y y1 = m(x x1)

1
y (1) = (x 2)
5
1
y + 1 = (x 2)
5
1
1
So, an equation is y + 2 = (x + 3) or y + 1 = (x 2).
5
5
1
y (2) = (x (3))
5
1
y + 2 = (x + 3)
5

40
41

4
3

6. m = =

y y1 = m(x x1)
4
y 0 = (x 1)
3
4
y = (x 1)
3

or

y y1 = m(x x1)
4
y 4 = (x 4)
3

4
4
So, an equation is y = (x 1) or y 4 = (x 4).
3
3

2. Sample answer: The slopes were the same, so the lines were

parallel. The constants in the point-slope form indicated the


graph was translated vertically and horizontally.

186

Algebra 1
Worked-Out Solutions

Copyright Big Ideas Learning, LLC


All rights reserved.

Chapter 4
7. Rewrite f (0) = 2 as (0, 2) and f (5) = 3 as (5, 3).

11. Line a: 2x + 6y = 12

2x 2x + 6y = 12 2x

3 2 5
m = = , or 1
50
5

6y = 12 2x

y y1 = m(x x1)

12 2x
6
1
1
y = 2 x, or y = x 2
3
3

6y
6

y 2 = 1(x 0)
y 2 = 1(x)
y 2 = x
y 2 + 2 = x + 2
y = x + 2

3
Line b: y = x 5
2
Line c: 3x 2y = 4
3x 3x 2y = 4 3x

So, a linear function is f (x) = x + 2.

2y = 4 3x
2y 4 3x
=
2
2
3
3
y = 2 + x, or y = x + 2
2
2

8. Rewrite f (1) = 6 as (1, 6) and f (4) = 6 as (4, 6).

6 (6) 6 + 6 0
m = = = , or 0
4 (1)
4+1
5
y y1 = m(x x1)
y (6) = 0(x 4)
y+6=0
66
y = 6

Lines b and c each have a slope of 32. So, they are parallel.
None of the lines have slopes that are negative reciprocals.
So, none are perpendicular.
12. Let (x1, y1) = (1, 1) and (x2, y2) = (2, 2).

y2 y1 2 (1) 2 + 1 3
m=
= = = , or 3
x2 x1
21
21 1

So, a linear function is f (x) = 6.


9. Rewrite f (3) = 2 as (3, 2) and f (2) = 3 as (2, 3).

3+2
5
3 (2)
m = = = , or 5
2 (3) 2 + 3 1
y = mx + b
2 = 5(3) + b
2 = 15 + b
+ 15

+ 15

13 = b
Using m = 5 and b = 13, a linear function is f (x) = 5x + 13.
1 2 1
12
2 (2) 2 + 2
4
0 (8) 0 + 8 8
Line b: m = = = , or 4
31
31 2
2 (3) 2 + 3 1
Line c: m = = =
0 (4)
0+4
4

10. Line a: m = = =

None of the lines have the same slope. So, none of the
1
lines are parallel. The slopes of lines a and b, 4 and 4
respectively, are negative reciprocals of each other. So, lines
aand b are perpendicular.

Copyright Big Ideas Learning, LLC


All rights reserved.

a.

y y1 = m(x x1)
y 2 = 3(x 6)
y 2 = 3(x) 3(6)
y 2 = 3x 18
+2

+2

y = 3x 16
So, an equation of the parallel line is y = 3x 16.
b. y y1 = m(x x1)

1
y 2 = (x 6)
3
1
1
y 2 = (x) (6)
3
3
1
y 2 = x + 2
3
+2
+2
1
y = x + 4
3
1
So, an equation of the perpendicular line is y = x + 4.
3

Algebra 1
Worked-Out Solutions

187

Chapter 4
13. Let (x1, y1) = (0, 2) and (x2, y2) = (2, 1).

y2 y1 1 2 1
m=
==
x2 x1 2 0
2
a.

y = mx + b
1
3 = (2) + b
2
3 = 1 + b
11

15. a. Words:

Number

of months

Linear model: C = 48 + 44 m
A linear model that represents this situation is C = 48 + 44m.

4 = b

b. C = 48 + 44m
1

= 48 + 44(6)
= 48 + 264
= 312

y = mx + b
3 = 2(2) + b

The total cost of setting up a website and maintaining it


for 6months is $312.

3 = 4 + b
+4

Initial
Charge
+ per month
fee

Variable: Let C be the total cost of setting up and


maintaining a website and m be the number of
months it is maintained.

Using m = 2 and b = 4, an equation of the parallel line


1
is y = 2x 4.
b.

Total
cost

+4

1=b
Using m = 2 and b = 1, an equation of the perpendicular
line is y = 2x + 1.
14. Let (x1, y1) = (3, 3) and (x2, y2) = (2, 1).

y2 y1
1 3
1 3 4
m=
= = = , or 4
x2 x1 2 (3) 2 + 3
1
a. y y1 = m(x x1)

y 0 = 4[x (4)]
y = 4(x + 4)
y = 4(x) 4(4)
y = 4x 16
So, an equation of the parallel line is y = 4x 16.
b. y y1 = m(x x1)

1
y 0 = [x (4)]
4
1
y = (x + 4)
4
1
1
y = (x) + (4)
4
4
1
y = x + 1
4
So, an equation of the perpendicular line is y = 14 x + 1.

c.

C = 48 + 44m

C = 62m

620 = 48 + 44m

620 = 62m
62m
620
=
62
62
10 = m

48 48
572 = 44m
572 44m
=
44
44
13 = m

At the original company, you can set up and maintain a


website for 13 months, but at the second company, you
can only set up and maintain a website for 10 months. So,
with $620, you can maintain the website for a longer time
at the original company.
150 155
10 8

5 145 150
2 12 10

5
2

140 145
14 12

5 130 140
2 16 14

5
2

16. = , = ,
= , =

w w1 = m(t t1)
5
w 150 = (t 10)
2
5
5
w 150 = (t) (10)
2
2
5
w 150 = t + 25
2
+ 150

+ 150

5
w = t + 175
2
Because the amount of water decreases at a constant rate,
the situation can be modeled by a linear equation. A linear
5
model is w = 2 t + 175.

188

Algebra 1
Worked-Out Solutions

Copyright Big Ideas Learning, LLC


All rights reserved.

Chapter 4
4.4 Explorations (p. 195)

2. a. Sample answer:

1. a. Sample answer:

Ages of American Women at First Marriage

Ages of Married Couples


y=x2
Age

y
80

Wifes age

75
70
65
60
55
50

Year

45
40
35
30
0

28
26
24
1
22
y = 8 x + 19.75
20
18
0
1960 1970 1980 1990 2000 2010 2020

0 30 35 40 45 50 55 60 65 70 75 80 x

Husbands age

Notice that as the husbands age increases, the wifes


age also increases at an approximately constant rate. So,
the trend can be modeled by a line with a positive slope.
Draw a line that appears to fit the data closely. Draw the
line through the middle of the points so that there are
approximately as many points above the line as below it.
Then write an equation using two points on the line such
as (40, 38) and (75, 73).
73 38 35
The slope of the line is m = = , or 1.
75 40 35
y y1 = m(x x1)
y 38 = 1(x 40)
y 38 = x 40
+ 38

+ 38

y=x2
An equation of a line that approximates the data is
y=x2.
b. Sample answer: Based on the equation, married couples are

usually close to the same age, but women are, on average,


a couple of years younger than their husband. The slope of
the line is 1 because their ages increase at the same rate.

Notice that the ages are increasing at an approximately


constant rate over time. So, the trend can be modeled by
a line with a positive slope. Draw a line that appears to
fit the data closely. Draw the line through the middle of
the points so that there are approximately as many points
above the line as below it. Then write an equation using
two points on the line such as (10, 21) and (50, 26).
5
1
26 21
The slope of the line is m = = , or .
50 10 40
8
y y1 = m(x x1)
1
y 21 = (x 10)
8
1
1
y 21 = (x) (10)
8
8
5
1
y 21 = x
4
8
+ 21

+ 21

1
y = x + 19.75
8
An equation of a line that approximates the data is
y = 18 x + 19.75.
1

b. Sample answer: The slope is 8 . So, every 8 years, the

median age of American women at their first marriage


increases by 1 year. The y-intercept is 19.75. So, in 1960
the median age of American women at their first marriage
was just under 20 years old.
1
8
1
y = (60) + 19.75
8

c. y = x + 19.75

y = 7.5 + 19.75
y = 27.25
So, if the trend continues, the median age of American
women at their first marriage in the year 2020 will be just
over 27 years old.
3. Sample answer: Scatter plots can show trends in data so that

you can identify any correlations between data sets. If the


data can be modeled by a line, then you can draw a line that
appears to fit the data closely. You can use two points on the
line to write an equation in slope-intercept form. So, you will
know the approximate rate of change, or slope, of the data
and an initial value if applicable.

Copyright Big Ideas Learning, LLC


All rights reserved.

Algebra 1
Worked-Out Solutions

189

4. Sample answer: The data in the graph is from the

Pennsylvania Department of Transportation.


PA Statewide Car Crashes

The scatter plot shows a negative correlation.


5. Sample answer:
Monthly Income vs. Monthly Car Payment
y

0 1 2 3 4 5 6 7 x

The number of crashes is decreasing at a fairly constant


rate over time. So, the data can be modeled by a line with
a negative slope. Estimate a line through the middle of the
plotted data, then use two points on that line to find the
equation. Use the points (6, 124.9) and (7, 122.8).
122.8 124.9 2.1
The slope of the line is m = = ,
76
1
or 2.1.
y y1 = m(x x1)
y 124.9 = 2.1 (x 6)
y 124.9 = 2.1x 2.1(6)
y 124.9 = 2.1x + 12.6
+ 124.9

+ 124.9

y = 2.1x + 137.5
An equation of a line that approximates the data is
y = 2.1x + 137.5.
4.4 Monitoring Progress (pp. 196198)
1. The smoothie has 260 calories.
2. The smoothie has 52 grams of sugar.
y
6

Attendance (thousands)

0 1 2 3 4 5 6 7 8 x

Age of car (years)

Years since 2005

3.

y
25
20
15
10
5
0

Monthly car payment


(dollars)

Crashes (thousands)

y
140
138
136
134
132
130
128
126
124
122
0

4.

Value (thousands
of dollars)

Chapter 4

y = 5x

440
400

(1900, 380)

360

(1700, 340)

320
280
0

0 1400 1600 1800 2000 2200

Monthly income (dollars)

Use (1700, 340) and (1900, 380).


40
1
380 340
The slope of the line is m = = = .
1900 1700 200 5
y y1 = m(x x1)
1
y 340 = (x 1700)
5
1
1
y 340 = (x) (1700)
5
5
1
y 340 = x 340
5
+ 340

+ 340

1
y = x
5
An equation of the line of fit is y = 15 x. The slope of the line is 15.
This means that a persons monthly car payment increases by
about $100 for every $500 increase in their monthly income, or
about $0.20 per dollar. The y-intercept is 0, meaning that a person
with no monthly income, has no car payment.

4.4 Exercises (pp.199200)

Vocabulary and Core Concept Check

1. When data show a positive correlation, the dependent

variable tends to increase as the independent variable


increases.

0 66 68 70 72 74 76 78 80 82 84 86 88 90 92 94 x

Temperature (F)

2. A line of fit is a line drawn on a scatter plot that is close to

most of the points of a data set.

The scatter plot shows a positive correlation.

190

Algebra 1
Worked-Out Solutions

Copyright Big Ideas Learning, LLC


All rights reserved.

Chapter 4
3. (16, 6)

4. (3, 14)

5. (7, 12)

6. (8, 17)

7. a. The laptop with a hard drive capacity of 8 gigabytes costs

$1100.
b. The $1200 laptop has a hard drive capacity of

12gigabytes.
c. The price tends to increase as the hard drive capacity

15. Sample answer:


World Birth Rates
Number of births
per 1000 people

Monitoring Progress and Modeling with Mathematics

increases.
8. a. The pitcher with an earned run average of 4.2 has a

winning percentage of 0.600.


b. The pitcher with a winning percentage of 0.33 has an

earned run average of 5.0.


c. The winning percentage tends to decrease as the earned

run average increases.


9. The y-values tend to increase as the x-values increase. So, the

scatter plot shows a positive correlation.


10. The points show no pattern. So, the scatter plot shows no

correlation.
11. The points show no pattern. So, the scatter plot shows no

correlation.
12. The y-values tend to decrease as the x-values increase. So,

the scatter plot shows a negative correlation.


y
4
3
2
1
0

(10, 32.5)
30

(40, 22.5)

20
0

40

a. Use (10, 32.5) and (40, 22.5).

1
22.5 32.5 10
The slope of the line is m = = , or .
3
40 10
30
y y1 = m(x x1)
1
y 32.5 = (x 10)
3
1
1
y 32.5 = (x) (10)
3
3
1
10
y 32.5 = x +
3
3
+ 32.5
+ 32.5
1
215
y = x +
3
6
1
215
An equation of the line of fit is y = x + .
3
6
1
b. The slope of the line is 3 . This means that every 3 years
the world birth rate decreases by 1 birth per 1000 people.
215
The y-intercept is
, or about 35.8, which is close to the
6
actual initial birth rate of 35.4 in 1960.

Working in Food Service


0 1 2 3 4 x

y
80

120
80

(0, 0)

40
0

60

y = 18x

(1, 18)
0

Hours worked

40

a. Use (0, 0) and (1, 18).

20
0

20

16. Sample answer:

The points show no pattern. So, the scatter plot shows no


correlation.
14.

215

y = 3x + 6

Years since 1960

Total earnings
(dollars)

13.

40

0 1 2 3 4 5 6 7 8 9 10 x

As the x-values increase, the y-values decrease. So, the


scatter plot shows a negative correlation.

18 0 18
The slope of the line is m = = , or 18.
10
1
y y1 = m(x x1)
y 0 = 18(x 0)
y = 18x
An equation of the line of fit is y = 18x.

b. The slope of the line is 18. This means that the server

earns about $18 per hour. The y-intercept is 0, which


means that if the server does not work any hours, then the
server will not make any money.

Copyright Big Ideas Learning, LLC


All rights reserved.

Algebra 1
Worked-Out Solutions

191

Chapter 4
17. Sample answer:

20. Sample answer: A car is going around the curve of an

High temperature
(degrees Fahrenheit)

32

43

50

65

73

80

Attendance at
exercise facility

100

95

83

72

63

55

I randomly chose 6 days out of the year. Then I compared


the high temperature in my town with the number of people
who went to a local indoor exercise facility to work out. The
data would probably show a negative correlation as with this
sample data set, because as the weather gets warmer, more
people exercise outside and fewer people go to the gym.

entrance ramp at 20 miles per hour. Then, as the ramp starts


to straighten out, the car begins to accelerate. For every
1second of acceleration, the cars speed increases by about
5miles per hour until the car merges with traffic on the
highway. The data could look something like this:
Elapsed time
(hours), x

Speed (miles
per hour), y

20 26 29 35 41 43 49 56 62 66

19. a. Sample answer:

Arm span (inches)

23. no; A line of fit is used to model a trend in the data and

should be close to most of the data points. If there is no


trend, it is not possible to draw a line that is close to most of
the points.

(65, 65)

24.

240
220
200
180
160
140
120
100
80
60
40
20

y=x
(50, 50)
0 45

line, such as (0, 10), (17, 135), (13, 107) and (21, 163).

75

45

22. Sample answer: Choose points that are on or close to the

55

relationship between two sets of numerical data, such as age


and time.

Height vs. Arm Span

65

21. Sample answer: A scatter plot is the best way to display a

18. no; Both the independent variable x and the dependent

variable y are decreasing. So, as you go backward through


the chart, both are increasing. Therefore, the data shows a
positive correlation.

55

65

75 x

Height (inches)

Use (50, 50) and (65, 65).

65 50 15
The slope of the line is m = = , or 1.
65 50 15
y y1 = m(x x1)
y 50 = 1(x 50)

18

y 50 = 1(x) 1(50)

The data follows a curve that decreases and then increases


(a parabola). The data cannot fit a line because the data
points do not have a linear trend.

y 50 = x 50
+ 50

+ 50

y=x
An equation of the line of fit is y = x.
b. The slope of the line is 1. This means that a persons arm

span increases by about 1inch for every 1inch increase


in height. The y-intercept is 0. The y-intercept has no
meaning in this context, because the height cannot be
0inches.

15

15

18 x

Maintaining Mathematical Proficiency


25.

g(x) = 6x
g(3) = 6(3)
= 18
When x = 3, g(x) = 18.
g(x) = 6x
g(0) = 6(0)
=0
When x = 0, g(x) = 0.
g(x) = 6x
g(4) = 6(4)
= 24
When x = 4, g(x) = 24.

192

Algebra 1
Worked-Out Solutions

Copyright Big Ideas Learning, LLC


All rights reserved.

Chapter 4
26.

h(x) = 10x
h(3) = 10(3)

4.5 Explorations (p. 201)


1. a. The ordered pair (25, 23.3) means that 25 years after

1960, or in 1985, the median age of American women at


their first marriage was 23.3.

= 30
When x = 3, h(x) = 30.

b. Check students work.

h(x) = 10x

c. Sample answer: An equation of the line of best fit is

h(0) = 10(0)

y = 0.13x + 19.8. The equation for Exploration 2 in


Section 4.4 was y = 18x + 19.75. The slope of this
equation, 18 = 0.125, is very close to the actual slope of
about 0.126 that was calculated by the linear regression
feature of a graphing calculator. The y-intercept, 19.75,
of the equation from Section 4.4 is also very close to the
calculated y-intercept, 19.845, for the line of best fit. So,
the equation from Section 4.4 is a close model for the
data, but not exactly the best fitting model.

=0
When x = 0, h(x) = 0.
h(x) = 10x
h(4) = 10(4)
= 40
When x = 4, h(x) = 40.
f (x) = 5x 8
f (3) = 5(3) 8

2. You can use a computer, spreadsheet, or the linear regression

feature of a graphing calculator to find an exact slope and


y-intercept of a line of best fit, or a line that minimizes the
distance from each data point to the line.

= 15 8
= 23
When x = 3, f (x) = 23.
f (x) = 5x 8
f (0) = 5(0) 8
=08
= 8
When x = 0, f (x) = 8.
f (x) = 5x 8
f (4) = 5(4) 8
= 20 8
= 12
When x = 4, f (x) = 12.
28.

v(x) = 14 3x
v(3) = 14 3(3)

3.

Temperature (F)

27.

y
95
90
85
80
75
70
65
13 14 15 16 17 18 19 20 21 x

Chirps per second

An equation of the line of best fit is y = 3.2746x + 24.984.


y = 3.2746x + 24.984
= 3.2746(19) + 24.984
= 62.2174 + 24.984
= 87.2014
So, when there are 19 chirps per second for striped ground
crickets, the outside temperature is about 87 F.

= 14 + 9
= 23
When x = 3, v(x) = 23.
v(x) = 14 3x
v(0) = 14 3(0)
= 14 0
= 14
When x = 0, v(x) = 14.
v(x) = 14 3x
v(4) = 14 3(4)
= 14 12
=2
When x = 4, v(x) = 2.

Copyright Big Ideas Learning, LLC


All rights reserved.

Algebra 1
Worked-Out Solutions

193

Chapter 4
4.5 Monitoring Progress (pp. 203205)

3. y = 9.6x + 844.5

1.

= 9.6(12) + 844.5
= 115.2 + 844.5

y-Value
from model

Residual

850

850

850 850 = 0

845

840.2

845 840.2 = 4.8

828

830.4

828 830.4 = 2.4

798

820.6

798 820.6 = 22.6

800

810.8

800 810.8 = 10.8

792

801

785

791.2

785 791.2 = 6.2

781

781.4

781 781.4 = 0.4

775

771.6

775 771.6 = 3.4

760

761.8

760 761.8 = 1.8

10

792 801 = 9

residual

10

= 729.3
So, in 2017, if the trend continues, the attendance at the
amusement park will be about 729,300 people.
4. Sample answer: There may be a correlation but no causal

relationship. Playing video games does not directly affect


your grade point average. However, it is likely that a student,
who spends a lot of time playing video games, will probably
spend less time studying and therefore have a lower grade
point average.
4.5 Exercises (pp. 206208)
Vocabulary and Core Concept Check
1. A residual is positive when the y-value of the actual data

point is greater than the y-value of the corresponding point


on the line of fit that has the same x-value. A residual is
negative when the y-value of the actual data point is less than
the y-value of the corresponding point on the line of fit that
has the same x-value.
2. A scatter plot of the residuals shows how well a model fits a

20

There are more points below the x-axis than there are above
the x-axis. So, the equation y = 9.8x + 850 does not model
the data well.
2. a. After entering the data from the table into two lists using

a graphing calculator, the linear regression feature yields


the equation y = 9.6x + 844.5.
870

data set. If the model is a good fit, then the absolute values of
the residuals are relatively small, and the residual points will
be more or less evenly dispersed about the horizontal axis. If
the model is not a good fit, then the residual points will form
some type of pattern that suggests the data are not linear.
3. Using a graph or its equation to approximate a value between

two known values is called interpolation. Extrapolation also


involves using a graph or its equation to approximate a value,
but extrapolation is predicting a value outside of the range of
known values.
4. The correlation coefficient r = 0.09 does not belong with

0
750

12

the other three, because this value is close to 0 and represents


a weak correlation, but the other three are close to either 1 or
1 and represent strong correlations.

b. The correlation coefficient is about 0.964. This means

that the relationship between the year and the attendance


has a strong negative correlation and the equation closely
models the data, as shown in the graph.
c. The slope of the line is 9.6. This means the attendance

at the amusement park decreases by about 9600people


each year. The y-intercept is 844.5. This means the
attendance in 2005 was 844,500 people.

194

Algebra 1
Worked-Out Solutions

Copyright Big Ideas Learning, LLC


All rights reserved.

Chapter 4
Monitoring Progress and Modeling with Mathematics

7.

5.

y-Value
from model

18

21

18 (21) = 3

13

17

13 (17) = 4

10

13

10 (13) = 3

7 (9) = 2

2 (5) = 3

0 (1) = 1

63=3

10

10 7 = 3

15

11

15 11 = 4

Residual

11.4

10

8.8

6.2

5 6.2 = 1.2

3.6

8 3.6 = 4.4

1 1 = 2

1.6

1 (1.6) = 2.6

4.2

4 (4.2) = 0.2

12

6.8

12 (6.8) = 5.2

9.4

7 (9.4) = 2.4

10 8.8 = 1.2

8 x

4
4

The points are evenly dispersed about the horizontal axis. So,
the equation y = 1.3x + 1 is a good fit.

4 x

8.

All of the residual values are above the horizontal axis. So,
the equation y = 4x 5 does not model the data well.

y-Value
from model

Residual

1 (4) = 3

3 (5) = 2

6 (6) = 0

10

8 (7) = 1

12

10

10 (8) = 2

14

10

10 (9) = 1

16

10

10

10 (10) = 0

18

11

9 (11) = 2

20

12

9 (12) = 3

6.

y-Value
from model

13

10

13 10 = 3

14

16

14 16 = 2

23

22

23 22 = 1

26

28

26 28 = 2

31

34

31 34 = 3

42

40

42 40 = 2

45

46

45 46 = 1

52

52

52 52 = 0

62

58

62 58 = 4

9 11.4 = 2.4

residual

Residual

residual

y-Value
from model

Residual

residual

residual
4

12

16

20 x

The residual points form a -shaped pattern, which suggests


the data are not linear. So, the equation y = 0.5x 2 does
not model the data well.

The points are evenly dispersed about the horizontal axis. So,
the equation y = 6x + 4 is a good fit.
Copyright Big Ideas Learning, LLC
All rights reserved.

Algebra 1
Worked-Out Solutions

195

Chapter 4
9.

13. After entering the data from the table into two lists using a

y-Value
from model

6.1

5.5

5.4

5.5 5.4 = 0.1

4.7

4.7

4.7 4.7 = 0

3.9

3.3

3.3

Residual
6 6.1 = 0.1

3.9 4 = 0.1
3.3 3.3 = 0

residual
0.1

graphing calculator, the linear regression feature yields the


equation y = 1.38x + 15.7. The correlation coefficient is
about 0.999. This means that the relationship between the
independent and dependent variables has a strong positive
correlation and the equation closely models the data.
14. After entering the data from the table into two lists using

a graphing calculator, the linear regression feature yields


the equation y = x + 11.25. The correlation coefficient is
about 0.444. This means that the relationship between the
independent and dependent variables has a weak negative
correlation and the equation does not model the data well.
15. The slope and y-intercept are reversed. Because the equation

is in the form y = ax + b, the slope is a = 4.47 and the


y-intercept is b = 23.16. An equation of the line of best fit is
y = 4.47x + 23.16.

0.1

16. Because the correlation coefficient is a negative number, the

The points are evenly dispersed about the horizontal axis.


So, the equation y = 0.7x + 6.8 is a good fit.

correlation is negative. The correlation coefficient is about


0.999. So, the data have a strong negative correlation.
17. a. After entering the data from the table into two lists using

10.

y-Value
from model

27

28.3

1.3

28

29.6

1.6

36

30.9

5.1

28

32.2

4.2

32

33.5

1.5

35

34.8

0.2

Residual

residual
4

a graphing calculator, the linear regression feature yields


the equation y = 381x 566.
2200

8
0

b. The correlation coefficient is about 0.989. This means that

there is a strong positive correlation between the number


of the minutes after an earthquake ended and the number
of people who reported the earthquake. So, the equation
closely models the data, as shown in the graph.
c. The slope of the line is 381. This means that the number

6 x

Most of the residual values are below the horizontal axis. So,
the equation y = 1.3x + 27 does not model the data well.
11. After entering the data from the table into two lists using

a graphing calculator, the linear regression feature yields


the equation y = 2.1x 7.5. The correlation coefficient is
about 0.980. This means that the relationship between the
independent and dependent variables has a strong positive
correlation and the equation closely models the data.

of people who reported an earthquake increased by


about 381 each minute after the earthquake ended. The
y-intercept is 565.7, which is meaningless in this
context. There cannot be a negative number of people.
18. a. After entering the data from the table into two lists using

a graphing calculator, the linear regression feature yields


the equation y = x + 7.
20

10
0

12. After entering the data from the table into two lists using a

graphing calculator, the linear regression feature yields the


equation y = 1.35x + 7.8. The correlation coefficient is
about 0.886. This means that the relationship between the
independent and dependent variables is a reasonably strong
negative correlation and the equation models the data well.

196

Algebra 1
Worked-Out Solutions

b. The correlation coefficient is about 0.619. This means that

the relationship between the day and the number of people


who volunteer has a weak positive correlation and the
equation does not model the data well.

Copyright Big Ideas Learning, LLC


All rights reserved.

Chapter 4
c. The slope of the line is 1. This means, the number of

volunteers increases by 1 each day. The y-intercept is


6.75, which has no meaning in this context because there
is no day 0.
19. a. After entering the data from the table into two lists using

a graphing calculator, the linear regression feature yields


the equation y = 0.18x + 19.7.
b. The correlation coefficient is about 0.968. This means

that the relationship between the mileage of a used


automobile and its selling price has a strong negative
correlation, and the equation y = 0.18x + 19.7 closely
models the data.
c. The slope of the line is 0.18. This means that the price

of a car decreases by about $180 for every thousand miles


the car has been driven. The y-intercept of the line is 19.7,
which has no meaning in this context because a used car
cannot have 0 mileage.
d.

y = 0.18x + 19.7
15.5 = 0.18x + 19.7
19.7

19.7

4.2 = 0.18x
0.18x
4.2
=
0.18
0.18
23.3 x
So, a car that costs $15,500 will have about 23,300 miles
on it.
e. y = 0.18x + 19.7

= 0.18 (6) + 19.7


= 1.08 + 19.7
= 18.62
So, a car with 6000 miles will cost about $18,620.
20. a. After entering the data from the table into two lists using

a graphing calculator, the linear regression feature yields


the equation y = 4.9x 37.7.
b. The correlation coefficient is about 0.936. This means

that the relationship between the lengths and costs of


the sailboats has a strong positive correlation, and the
equation y = 4.9x 37.7 closely models the data.
c. The slope of the line is 4.9. This means that the cost of a

sailboat increases by about $4900 per foot of a sailboats


length. The y-intercept of the line is 37.7. However,
the y-intercept is meaningless in this context because
a sailboat cannot be 0 feet long, and the cost cannot be
negative.

e.

y = 4.9x 37.7
147 = 4.9x 37.7
+ 37.7

+ 37.7

184.7 = 4.9x
37.7 x
So, a sailboat that costs $147,000 is about 37.7 feet long.
21. There is a negative correlation and a causal relationship

because the more time you spend talking on a cell phone, the
more you will drain the remaining battery life.
22. There may be a positive correlation but no causal

relationship. Growing taller will not cause a toddler to learn


new words, but as a toddler grows, he or she will probably be
learning more words.
23. There is no correlation between the number of hats you own

and the size of your head because buying more hats will not
change the size of your head.
24. There may be a positive correlation but no causal

relationship. The weight of a dog does not determine the


length of its tail.
25. Sample answer: Each week of the summer season, a shop

at the beach recorded the numbers x of ice cream cones sold


and the numbers y of surf board rentals. This data set has a
strong positive correlation but no causal relationship. Selling
more ice cream cones will not cause the number of surf
board rentals to increase. However, on the days with good
weather in the height of the season, both numbers will likely
be high, and on days with bad weather, both numbers will
likely be low.
26. a. B; The correlation coefficient is r = 0.98 because the

graph shows data with a strong positive correction that is


closely modeled by the line.
b. C; The correlation coefficient is r = 0.97 because the

graph shows data with a strong negative correlation that is


closely modeled by the line.
c. A; The correlation coefficient is r = 0 because the graph

shows data with no correlation.


d. D; The correlation coefficient is r = 0.69 because the

graph shows data with a weak positive correlation that is


roughly modeled by the line.

d. y = 4.9x 37.7

= 4.9(20) 37.7
= 98 37.7
= 60.3
So, a sailboat that is 20 feet long will cost about $60,300.

Copyright Big Ideas Learning, LLC


All rights reserved.

Algebra 1
Worked-Out Solutions

197

Chapter 4
using a graphing calculator, the linear regression feature
yields the equation y = 0.08x + 3.8. The correlation
coefficient is about 0.965. This means that the
relationship between grade point average and hours spent
watching television has a strong negative correlation.
b. The slope of the line is 0.08. This means that a students

grade point average decreases by about 0.08 for every


hour spent watching television each week. The y-intercept
of the line is 3.8. This means that a student who watches
no television is likely to have a grade point average of
about 3.8.
c. y = 0.08x + 3.8

= 0.08 (14) + 3.8


= 1.12 + 3.8
= 2.68
So, a student who watches 14 hours of television each
week will have a grade point average of about 2.68.
d. Sample answer: no; Watching a lot of television does not

directly cause a student to get lower grades. However,


students who spend a lot of time watching television will
probably spend less time studying and therefore earn
lower grades.

30. Sample answer:


Beach Attendance for One Week
y

Beach 2

27 a. After entering the data from the table into two lists

yes; The model predicts that a student who watches 2hours


of television will have a grade point average of about 3.64.
Because the actual grade point average, 2.4, is significantly
less, it will weaken the correlation between the model and
the actual data.
y = 381x 566

= 381(9) 566

= 381(15) 566

= 3429 566

= 5715 566

= 2863

= 5149

So, using the equation y = 381x 566 of the model


from Exercise 17, the number of people who reported
the earthquake 9 minutes after in ended was about 2863
and the number of people who reported the earthquake
15minutes after it ended was about 5149.
b. The difference between the first extrapolation and the

actual data value is only 113. The difference between the


second extrapolation and the actual data value is 1949.
So, the second prediction was less accurate than the
first. This fits with what you would expect because the
farther removed a value is from the known values, the less
confidence you can have in the accuracy of the prediction.
Sometimes trends change over time.

Algebra 1
Worked-Out Solutions

20

40

60

80

Beach 1

As the number of people at Beach1 increases, the number


of people at Beach2 also increases. A possible correlation
coefficient is about 0.9 because there is a strong positive
correlation between the number of people at the two beaches,
and the graph shows that the points could fit closely to a line.
This does not represent a causal relationship, however. The
factors that affect how many people come to the beaches are
things such as the temperature outside and whether it is a
weekend or holiday.
31. a. After entering the data from the table into two lists

using a graphing calculator, the linear regression feature


yields the equation y = 513.5x 298. The correlation
coefficient is about 0.993. This means that the relationship
between the year and the numbers of the messages sent
has a strong positive correlation and the equation closely
models the data.
does not just go up simply because another year has
passed. The number of text messages increases each year
because cell phone service gets faster, more efficient,
and less expensive over time as the cell phone companies
work to improve their services.

= 3.64

198

b. no; Sample answer: The number of text messages sent

= 0.16 + 3.8

29. a. y = 381x 566

40
0

28. y = 0.08x + 3.8

= 0.08(2) + 3.8

80

c.

y-Value from
model

241

215.5

601

729

1360

1242.5

1806

1756

2206

2269.5

Residual
241 215.5 = 25.5
601 729 = 128
1360 1242.5 = 117.5
1806 1756 = 50
2206 2269.5 = 63.5

residual
100

100

The points are evenly dispersed about the horizontal axis.


So, the equation y = 513.5x 298 is a good fit.

Copyright Big Ideas Learning, LLC


All rights reserved.

Chapter 4
d. Sample answer: Using the graphing calculator to calculate

a correlation coefficient is faster than calculating and


graphing the residuals. Both methods will reveal how
closely a model fits the actual data. The graph of the
residuals sometimes reveals why a linear model is not a
good fit, if the data fits a nonlinear model for instance.
The correlation coefficient, however, will simply reveal
that the linear model is not a good fit but does not help
determine why.

2. An arithmetic sequence can describe a pattern in which

the difference between consecutive terms is the same. For


example, if every time you beat a level of a game, you earn
50 points.
3. The number y of atoms increases by 3 for each additional

water molecule. Each y-value is 3 times the corresponding


x-value. So, 23 molecules of water have 3(23) = 69 atoms.
4.6 Monitoring Progress (pp. 210213)

Maintaining Mathematical Proficiency


32. nonlinear; As x increases by 1, the y-value alternates between

1. Position

4 and 4. The rate of change is not constant. So, the


function is not linear.
change is constant. So, the function is linear.

2.

4.6 Explorations (p. 209)


1. a. Number of stars, n

Number of sides, y

10

20

30

40

50

Number of circles, y

1
2

2
3

3
4

Position
Term

1
0.2

3. Position

Term

1
2

2
4

3
6

4
5

4
8

6
48

7
60

5
1.8

6
2.2

7
2.6

+12 +12 +12

2
0.6

3
1

4
1.4

2
3
3
4

3
1
3
2

4
1
3
4

5
6

The next three terms are 3,


4.

7
1
2
2

2 34 ,

and 2 12 .

Position, n

Term, an

12

an
16
14
12
10
8
6
4
2
0

(4, 12)
(3, 9)
(2, 6)
(1, 3)
0 1 2 3 4 5 6 7 n

The points lie on a line.


5.

The y-values increase by 2 each time a row is added. So,


each y-value is twice the corresponding value of n.

6
3
2
4

( ) ( ) ( )

5
10

0 1 2 3 4 5 n

1
1
1
+ + +
4
4
4

0 1 2 3 4 5 n

Number of rows, n
Number of dots, y

5
36

The next three terms are 1.8, 2.2, and 2.6.

The y-values increase by 1. So, each y-value is 1 more


than the corresponding value of n.

y
12
10
8
6
4
2
0

4
24

0 1 2 3 4 5 n

b. n

c.

3
12

+0.4 +0.4 +0.4

The y-values increase by 10. So, each y-value is 10 times


the corresponding value of n.

y
6
5
4
3
2
1
0

2
0

The next three terms are 36, 48, and 60.

33. linear; As x increases by 2, y decreases by 5. The rate of

y
60
50
40
30
20
10
0

1
12

Term

Position, n

Term, an

5
4
3
2
1
1
2
3

an

(1, 4)
(2, 2)
n

(3, 0)
1 2 3 4 5 6 7

(4, 2)

The points lie on a line.


Copyright Big Ideas Learning, LLC
All rights reserved.

Algebra 1
Worked-Out Solutions

199

Chapter 4
6. Position, n

Term, an

2
0.8

3
0.6

4
0.4

11. a. f (n) = a1 + (n 1)d

f (n) = 7 + (n 1)(2)
f (n) = 7 + n(2) 1(2)
f (n) = 7 + 2n 2
f (n) = 2n + 5

(1, 1)
(2, 0.8)
(3, 0.6)
(4, 0.4)

The function f (n) = 2n + 5 represents the arithmetic


sequence.
b.

0 1 2 3 4 5 6 7 n

The points lie on a line.


7. Position, n

Term, an

1
2

2
4

3
7

4
11

+2
+3
+4

Consecutive terms do not have a common difference. So, the


sequence of terms 2, 4, 7, 11,... is not arithmetic.
8. an = a1 + (n 1)d

an = 4 + (n 1)(1)
an = 4 + n(1) 1(1)
an = 4 + n 1
an = n + 3
An equation for the nth term is an = n + 3.
an = n + 3
a25 = 25 + 3
= 28
The 25th term of the arithmetic sequence is 28.
9. an = a1 + (n 1)d

an = 8 + (n 1)(8)
an = 8 + n(8) 1(8)
an = 8 + 8n 8
an = 8n
An equation for the nth term is an = 8n.
an = 8n

Total cost (dollars)

an
1.6
1.4
1.2
1.0
0.8
0.6
0.4
0.2
0

1
1

an
16
14
12
10
8
6
4
2
0

0 1 2 3 4 5 6 7 8 n

Games

c. f (n) = 2n + 5

29 = 2n + 5
5

12 = n
You can play 12 games.
4.6 Exercises (pp. 214216)
Vocabulary and Core Concept Check
1. The graph of an arithmetic sequence is a graph of a linear

function whose domain is the set of positive integers.


2. The one that is different is find the difference between the

terms a2 and a4. This answer is 16 10 = 6. The answer of


all the others is 13 10 = 3.
Monitoring Progress and Modeling with Mathematics
3. Position

Term

a25 = 8(25)
= 200
The 25th term of the arithmetic sequence is 200.
10. an = a1 + (n 1)d

an = 1 + (n 1)(1)
an = 1 + n(1) 1(1)
an = 1 n + 1
an = n + 2
An equation for the nth term is an = n + 2.
an = n + 2
a25 = 25 + 2
= 23

24 = 2n
24 2n
=
2
2

1
2

2
15

3
28

4
41

+13
+13
+13

The next three terms are 15, 28, and 41.


4. Position

Term

1
18

2
12

3
6

4
0

+(6) +(6) +(6)


The next three terms are 12, 6, and 0.
5. 18 13 = 5, 23 18 = 5, 28 23 = 5

The common difference is 5.


6. 150 175 = 25, 125 150 = 25, 100 125 = 25

The common difference is 25.

The 25th term of the arithmetic sequence is 23.

200

Algebra 1
Worked-Out Solutions

Copyright Big Ideas Learning, LLC


All rights reserved.

Chapter 4
17. Position, n

7. 12 (16) = 12 + 16 = 4,

8 (12) = 8 + 12 = 4,
4 (8) = 4 + 8 = 4

an
32
28
24
20
16
12
8
4
0

The common difference is 4.


1 1
1
2
3
2
2
8. 3 4 = 3 3 = , 3 3 = ,
3 3
3
3
3
3
3
1
3
4
1
3 3 = 2 2 =
3
3
3
3
1
The common difference is .
3
9. 5 6.5 = 1.5, 3.5 5 = 1.5, 2 3.5 = 1.5

2
22

3
25

4
28

5
31

6
34

7
37


+3

+3

+3

2
0

3
15

4
30

2
3

3
5

4
7

2
19

3
36

4
53

(1, 4)
0 1 2 3 4 5 6 7 n

50
40
30
20
10

The common difference is 9.


1
19

4
28

(2, 12)

11 2 = 9

Position
Term

3
20

(3, 20)

1
15

Term, an

10. 7 (16) = 7 + 16 = 9, 2 (7) = 2 + 7 = 9,

2
12

(4, 28)

18. Position, n

The common difference is 1.5.

11.

1
4

Term, an

10
20
30

an

(4, 30)
(3, 15)
(2, 0)
2 3 4 5 6 7 n

(1, 15)

The next three terms are 31, 34, and 37.


12. Position

1
1

Term

2
12

3
23

4
34

5
45

6
56

7
67

+11

+11 +11
The next three terms are 45, 56, and 67.
13. Position

1
16

Term

2
21

3
26

4
31

5
36

6
41

7
46


+5

+5

+5

The next three terms are 36, 41, and 46.


14. Position

Term

1
60

2
30

3
0

4
30

5
60

6
90

7
120

+(30) +(30) +(30)


The next three terms are 60, 90, and 120.
15. Position

Term

1
1.3

2
1

3
0.7

4
0.4

5
0.1

6
0.2

7
0.5

+(0.3) +(0.3) +(0.3)


The next three terms are 0.1, 0.2, and 0.5.
16. Position

Term

1
5

2
2

3
1

4
1

5
1

6
0

19. Position, n

1
1

Term, an
0
1
2
3
4
5
6
7
8
an

0 1 2 3 4 5 6 7 n

(1, 1)
(2, 3)
(3, 5)
(4, 7)

20. Position, n

1
2

Term, an
an
80
70
60
50
40
30
20
10
0

(4, 53)
(3, 36)
(2, 19)
(1, 2)
0 1 2 3 4 5 6 7 n

7
1

( ) ( ) +( 16 )

1
1
+ +
6
6

The next three terms are 16, 0, and 6.


Copyright Big Ideas Learning, LLC
All rights reserved.

Algebra 1
Worked-Out Solutions

201

Chapter 4
21. Position, n

Term, an
an
16
14
12
10
8
6
4
2
0

(4,

1
13 2

1
13
2

28. Position

Term

1
5

2
9

3
14

+4
+5
+6

4
20

Consecutive terms do not have a common difference.


So, the sequence is not arithmetic.

29. Position

(3, 9)

Term

(2, 4 12)

1
48

2
24

3
12

4
6

+(24) +(12) +(6)

(1, 0)

Consecutive terms do not have a common difference.


So, the sequence is not arithmetic.

0 1 2 3 4 5 6 7 n

22. Position, n

1
6

Term, an
an
8
7
6
5
4
3
2
1
0

2
1
4
2

2
5.25

3
4.5

4
3.75

30. Position

Term

1
87

2
81

3
75

4
69

+(6) +(6) +(6)


(1, 6)

The sequence is arithmetic with a common difference of 6.

(2, 5.25)
(3, 4.5)

31. Position

(4, 3.75)

Term
0 1 2 3 4 5 6 7 8 n

23. The points do not lie on a line. So, the sequence 1, 4, 1,

4,... is not arithmetic.


24. Position, n

Term, an

1
5

3
19

Term

4
26

+7

+7

+7

Term, an

1
70

2
55

3
40

4
25


+(15) +(15) +(15)

Consecutive terms have a common difference of 15.


So, the graph represents the arithmetic sequence 70, 55, 40,
25, . . . .
26. Position, n

Term, an

1
2

2
10

3
16

4
20


+8

+6

+4

Consecutive terms do not have a common difference, and the


points do not lie on a line. So, the sequence 2, 10, 16, 20, . . .
is not arithmetic.
27. Position

Term

1
13

2
26

3
39

+13
+13
+13

3
8

+2
+2
+2

1
2

2
5

3
8


+3

Consecutive terms have a common difference of 7. So, the


graph represents the arithmetic sequence 5, 12, 19, 26, ....
25. Position, n

2
6

4
10

Consecutive terms have a common difference of 2.


So, the sequence 4, 6, 8, 10, . . . is arithmetic.
32. Position

2
12

1
4

+3

4
11

+3

Consecutive terms have a common difference of 3.


So, the sequence 2, 5, 8, 11, . . . is arithmetic.
33. d = 4 (5) = 4 + 5 = 1

an = a1 + (n 1)d
an = 5 + (n 1)(1)
an = 5 + n(1) 1(1)
an = 5 + n 1
an = n 6
An equation for the nth term of the arithmetic sequence
is an = n 6.
an = n 6
a10 = 10 6
=4
The 10th term of the arithmetic sequence is 4.

4
52

The sequence is arithmetic with a common difference of 13.

202

Algebra 1
Worked-Out Solutions

Copyright Big Ideas Learning, LLC


All rights reserved.

Chapter 4
34. d = 9 (6) = 9 + 6 = 3

37. d = 0 10 = 10

an = a1 + (n 1)d

an = a1 + (n 1)d

an = 6 + (n 1)(3)

an = 10 + (n 1)(10)

an = 6 + n(3) 1(3)

an = 10 + n(10) 1(10)

an = 6 3n + 3

an = 10 10n + 10

an = 3n 3

an = 10n + 20

An equation for the nth term of the arithmetic sequence


is an = 3n 3.

An equation for the nth term of the arithmetic sequence


is an = 10n + 20.

an = 3n 3

an = 10n + 20

a10 = 3(10) 3

a10 = 10(10) + 20

= 30 3

= 100 + 20

= 33

= 80

The 10th term of the arithmetic sequence is 33.


1
2

The 10th term of the arithmetic sequence is 80.


4
7

1
2

3
7

1
7

38. d = =

35. d = 1 =

an = a1 + (n 1)d

an = a1 + (n 1)d

()
1
1
1
a = + n ( ) 1( )
2
2
2

3
1
an = + (n 1)
7
7

1
1 1
an = + n
2
2 2

1
3 1
an = + n
7
7 7

1
an = n
2

2
1
an = n +
7
7

An equation for the nth term of the arithmetic sequence


is an = 12 n.

An equation for the nth term of the arithmetic sequence


is an = 17 n + 27 .

1
an = n
2
1
a10 = (10)
2

2
1
an = n +
7
7
2
1
a10 = (10) +
7
7
10 2
=+
7
7
5
12
= = 1
7
7

1
1
an = + (n 1)
2
2
n

=5
The 10th term of the arithmetic sequence is 5.
36. d = 110 100 = 10

an = a1 + (n 1)d
an = 100 + (n 1)(10)
an = 100 + n(10) 1(10)
an = 100 + 10n 10
an = 10n + 90
An equation for the nth term of the arithmetic sequence
is an = 10n + 90.
an = 10n + 90

()
3
1
1
a = + n ( ) 1( )
7
7
7
n

The 10th term of the arithmetic sequence is 157.


39. Because consecutive terms are decreasing by 1, the common

difference should be negative.


2,

1,

0,

1, . . .

+(1) +(1) +(1)


The common difference is 1.

a10 = 10(10) + 90
= 100 + 90
= 190
The 10th term of the arithmetic sequence is 190.

Copyright Big Ideas Learning, LLC


All rights reserved.

Algebra 1
Worked-Out Solutions

203

Chapter 4
40. The equation an = a1 + (n 1)d should be used to write an

44. a. The next three figures are:

equation for the nth term of the arithmetic sequence.


14, 22, 30, 38, . . .
d = 22 14 = 8
an = a1 + (n 1)d
an = 14 + (n 1)(8)
an = 14 + n(8) 1(8)

b. The 20th figure in the sequence is a circle with 40 equal

sections.

an = 14 + 8n 8
an = 8n + 6

45. a. d = 10 5 = 5

An equation for the nth term of the arithmetic sequence is


an = 8n + 6.

f (n) = a1 + (n 1)d
f (n) = 5 + (n 1)(5)
f (n) = 5 + n(5) 1(5)

41. a1 = 3

f (n) = 5 + 5n 5

d = 1.5(3) = 4.5
Position, n
Term, an

1
3

2
7.5

3
12

f (n) = 5n

4
16.5

A function that represents the arithmetic sequence is


f (n) = 5n.

The next three terms are 7.5, 12, and 16.5.

b.
Total babies born

an
24
21
18
15
12
9
6
3
0

(4, 16.5)
(3, 12)
(2, 7.5)
(1, 3)

Term, an

1
10

2
12

3
14

4
16

5
18

The first five terms are 10, 12, 14, 16, and 18.
an
24
21
18
15
12
9
6
3
0

(2, 12)
(1, 10)

(4, 20)
(3, 15)
(2, 10)
(1, 5)
0 1 2 3 4 5 6 7 8 n

Minutes after Midnight


January 1st

0 1 2 3 4 5 6 7 8 n

42. Position, n

an
24
21
18
15
12
9
6
3
0

(5, 18)
(4, 16)
(3, 14)

c. f (n) = 5n

100 = 5n
100 5n
5
5
20 = n

It takes about 20 minutes for 100 babies to be born.

0 1 2 3 4 5 6 7 8 n

43. a. The next three figures are:

b. The 20th figure in the sequence is a regular polygon with

22 sides.

204

Algebra 1
Worked-Out Solutions

Copyright Big Ideas Learning, LLC


All rights reserved.

Chapter 4
51. Sample answer:

46. a. d = 40 48 = 8

an = a1 + (n 1)d

Position
Term

an = 56 + (n 1)(8)

1
6

an = 56 + n (8) 1(8)

A function that represents the arithmetic sequence is


an = 8n + 64.
an = 8n + 64

an = 6 + n(3) 1(3)
an = 6 3n + 3
Position
Term

64

48 = 8n

1
5

4
14

Another arithmetic sequence


with a common difference
of3 is 5, 8, 11,
14, . . .. An equation
forthe nth term is
an = 3n 2.

an = 5 + (n 1)(3)

The movie earns $16 million in week 6.


c. 56 + 48 + 40 + 32 + 24 + 16 + 8 = 224

The movie earns $224 million overall.


1
1

2
5

3
9


+4

+4

f (n) = a1 + (n 1)d
f (n) = 1 + (n 1)(4)
f (n) = 1 + n(4) 1(4)

an = 5 + n(3) 1(3)
an = 5 3n + 3
an = 3n 2

52. Sample answer: An algebra class had 22 students at the

beginning of the term, and then after the first week, one
new student joined the class each week for 5 more weeks.
The sequence of how many students were in the class at the
end of each of the first 6 weeks, 22, 23, 24, 25, 26, 27, is
arithmetic with a common difference of 1.
53. 20 + 19 + 18 + 17 + 16 + 15 + 14 = 119

f (n) = 1 + 4n 4

The pile has 119 logs in total.

f (n) = 4n 3

54. a. Size

f (30) = 4(30) 3

Cost

= 120 3

1
20,000

= 117

2
32,500

+12,500

Because the areas of consecutive figures have a common


difference of 4, the areas form an arithmetic sequence 1, 5,
9, . . . . A function that represents the arithmetic sequence is
f (n) = 4n 3. The 30th figure has an area of 117 square
inches.

Term, an

3
11

an = a1 + (n 1)d

6=n

48. Position, n

2
8

+(3) +(3) +(3)

48 8n
=
8
8

Term, an

An arithmetic sequence
witha common difference
of3 is 6, 3, 0, 3, . . . . An
equation for the nth term is
an = 3n + 9.

an = 6 + (n 1)(3)

an = 3n + 9

16 = 8n + 64

47. Position, n

4
3

an = a1 + (n 1)d

an = 8n + 64

64

3
0

+(3) +(3) +(3)

an = 56 8n + 8

b.

2
3

1
4

2
9

3
25

+5

+16

Because the sequence of areas 4, 9, 25, . . . does not have a


common difference, the sequence is not arithmetic.
49. The domain and range of an arithmetic sequence are both

discrete because they consist of individual numbers, not a


range of numbers.

3
45,000

4
57,500

+12,500 +12,500

yes; Consecutive terms have a common difference of


$12,500. So, the sequence 20,000, 32,500, 45,000,
57,500,... is arithmetic.
b. Sample answer: You could write an equation for the nth

term in the sequence and then find a6.


an = a1 + (n 1)d

an = 12,500n 7500

an = 20,000 + (n 1)(12,500)

a6 = 12,500(6) 7500

an = 20,000 + n(12,500) 1(12,500)


an = 20,000 + 12,500n 12,500
an = 12,500n 7500
An equation of the nth term
of the arithmetic sequence is
an = 12,500n 7500.

= 75,000 7500
= 67,500
A six-page
advertisement would
cost $67,500.

50. no; If the first term of the sequence is negative and the output

values constantly increase, or if the first term of the sequence


is positive and the output values constantly decrease, the
range contains positive and negative numbers.
Copyright Big Ideas Learning, LLC
All rights reserved.

Algebra 1
Worked-Out Solutions

205

Chapter 4
41 23
4 1

18
3

Maintaining Mathematical Proficiency

55. d = = = 6

Position
Term

58. x + 8 9

1
23

2
29

3
35


+6

+6

4
41

x 17
The solution is x 17.

+6

f (n) = a1 + (n 1)d

30 25 20 15 10

f (n) = 23 + (n 1)(6)
f (n) = 23 + n (6) 1(6)

+4

f (n) = 6n + 17

10

10

12

14

16

y 4

62. x

h(x)

2
6
8
7
6
5
4
3

= 3x + 1 x 1

= 6x

30

The solution is y 4.

b. 3x + 1 (x + 1)

= 9x + 1 3x 1

25

yes; Because consecutive terms have a common difference


of 2x, the sequence is arithmetic.

9x + 1 (3x + 1)

20

+ 21

= 2x

= 2x

10

7+y3

61.

= 2x

= 7x + 6 5x 6

t<9

You must wait 7 minutes for the train.

7x + 6 (5x + 6)

The solution is t < 9.

7:36 7:29 = :07

= 5x + 6 3x 6

15

+4

+ 21

+12 +12 +12 +12

= 2x

10

60. t 21 < 12

+12 +12 +12 +12

5x + 6 (3x + 6)

The solution is b > 19.

56.

= 3x + 6 x 6

19 < b

A function that represents the arithmetic sequence is


f (n) = 6n + 17.

57. a. 3x + 6 (x + 6)

15 < b 4

59.

f (n) = 23 + 6n 6

Stop
1
2
3
4
5
6
7
8
9
number
Time
6:00 6:12 6:24 6:36 6:48 7:00 7:12 7:24 7:36

4 3 21

1
3

0
0

1
3

2
6

h(x) = 3
x

1 2 3 4 x

The function h is of the form y = a f (x), where a = 3. So, the


graph of h is a vertical stretch of the graph by a factor of 3.
The domain is all real numbers. The range is y 0.

27x + 1 (9x + 1)
= 27x + 1 9x 1
= 18x
no; Because consecutive terms do not have a common
difference, the sequence is not arithmetic.

206

Algebra 1
Worked-Out Solutions

Copyright Big Ideas Learning, LLC


All rights reserved.

Chapter 4
63. x

3
2

v(x)
8
7
6
5
4
3

4
1

5
0

6
1

7
1

1. a. yes; No vertical line can be drawn through more than one

point on the graph. So, each x-value is paired with exactly


one y-value, and the graph represents y as a function of x.

b. If x = 0, then y = 0 because there is a closed dot at (0, 0).


c. Use (2, 2) and (1, 1).

v(x) =
x 5

12
1
12
m = = = , or 1
1 (2) 1 + 2
1
y y1 = m(x x1)

2
1

y 1 = 1(x (1))

1 2 3 4 5 6 7 8 x

The function v is of the form y = f (x h), where h = 5.


So, the graph of v is a horizontal translation 5 units right of
the graph of f.
The domain is all real numbers. The range is y 0.
64.

4.7 Explorations (p. 217)

8
7
6
5
4
3

y 1 = 1(x) 1(1)
y 1 = x 1
+1

g(x)

y 1 = 1(x + 1)

+1

y = x
So, f (x) = x, if x 0.
d. Use (1, 2) and (2, 2).

22 0
m = = , or 0
21 1
y y1 = m(x x1)

g(x) =
x
+ 1

y 2 = 0(x 1)
y2=0
+2

4 3 21

y=2

1 2 3 4 x

The function g is of the form y = f (x) + k, where k = 1.


So, the graph of g is a vertical translation 1 unit up of the
graph of f.
The domain is all real numbers. The range is y 1.
65.

r(x)

3
4
5
6
7
8

So, f (x) = 2, if x > 0.


e. f (x) =

{ x,2,

if x 0
if x > 0

2. a. yes; No vertical line can be drawn through more than one

4 3 21

+2

1 2 3 4 x

r(x) = 2
x

point on the graph. So, each x-value is paired with exactly


one y-value, and the graph represents y as a function of x.

2, if 6 x < 3
0,
if 3 x < 0
b. f (x) =
2,
if 0 x < 3
4,
if 3 x < 6
3. Sample answer: In order to describe a function that is

represented by more than one equation, you can write an


equation for each portion and give the range of domain
values for which the equation describes the graph. Combine
all of the equations and corresponding ranges of domain
values into one function with a bracket { symbol.

The function r is of the form y = a f (x), where a = 2. So,


the graph of r is a vertical stretch of the graph of f by a factor
of 2 and a reflection in the x-axis.
The domain is all real numbers. The range is y 0.

Copyright Big Ideas Learning, LLC


All rights reserved.

Algebra 1
Worked-Out Solutions

207

Chapter 4
4. For x 0, use (2, 2) and (1, 1).

7.

12
1
12
m = = = , or 1
1 (2) 1 + 2
1
y y1 = m(x x1)

Closed Open
x

y 2 = 1[x (2)]

2
1

y 2 = 1(x + 2)
y 2 = 1(x) 1(2)

4 3

1 2 3 4 x

y 2 = x 2
+2

2
3
4

+2

y = x
For x > 0, use (1, 1) and (2, 2).
21 1
m = = , or 1
21 1
y y1 = m(x x1)
y 1 = 1(x 1)
y 1 = 1(x) 1(1)
y1=x1
+1

5
6

The domain is all real numbers. The range is y 1.


8.

Open Closed
x

0.5

+1

8
6

y=x
So, the graph can be represented by the function
x, if x 0
.
f (x) =
x, if x > 0

4 3 21

The value of f is 3 when x = 8.


2.

f (x) = x + 2
f (2) = 2 + 2
f (2) = 0
The value of f is 0 when x = 2.

3. f (x) = x + 2

f (0) = 0 + 2
f (0) = 2
The value of f is 2 when x = 0.
4. f (x) = x + 2

f (3) = 3 + 2
f (3) = 5
The value of f is 5 when x = 3.
5. f (x) = x + 2

f (5) = 5 + 2
f (5) = 7
The value of f is 7 when x = 5.
6. f (x) = 4x

f (10) = 4(10)
f (10) = 40
The value of f is 40 when x = 10.

208

Algebra 1
Worked-Out Solutions

1 2 3 4 x

4
6
8

f (x) = 3
f (8) = 3

4
2

4.7 Monitoring Progress (pp. 218221)


1.

The domain is all real numbers. The range is y < 2 or y 0.


9. For x 0, use (2, 1) and (1, 0).

01
1
01
m = = = , or 1
1 (2) 1 + 2
1
y y1 = m(x x1)
y 0 = 1[x (1)]
y = 1(x + 1)
y = 1(x) 1(1)
y = x 1
For x > 0, use (1, 3) and (2, 4).
43 1
m = = , or 1
21 1
y y1 = m(x x1)
y 3 = 1(x 1)
y 3 = 1(x) 1(1)
y3=x1
+3

+3

y=x+2
So, a piecewise function for the graph is
x 1, if x 0
f (x) =
.
x + 2, if x > 0

Copyright Big Ideas Learning, LLC


All rights reserved.

Chapter 4
10. For x 2, use (4, 2) and (3, 1).

12
1
12
m = = = , or 1
3 (4) 3 + 4
1

g(x) = a x 5 + 4

0 = a3 5 + 4

y y1 = m(x x1)

0 = a 2 + 4

y 2 = 1[x (4)]

0 = a(2) + 4

y 2 = 1(x + 4)

0 = 2a + 4

y 2 = 1(x) 1(4)

y 2 = x 4

4 = 2a
4 2a
=
2
2
2 = a

+2

+2

y = x 2
For 2 < x < 1, y = 2.

b. g(x) = 2 x 5 + 4

g(x) =

y 1 = 2(x 2)

g(x) = 2(x) 2(5) + 4,

y = 2x 3
So, a piecewise function for the graph is
x 2, if x 2
f (x) = 2,
if 2 < x < 1.
2x 3, if x 1

Number of
days

Total cost
(dollars)

0<x1

100

1<x2

150

2<x3

200

3<x4

250

g(x) = 2x 10 + 4,
g(x) = 2x 6,

if x < 5

g(x) = 2(x 5) + 4, if x 5 0
+5+5
g(x) = 2(x) 2(5) + 4,
g(x) = 2x + 10 + 4,
g(x) = 2x + 14,

if x 5

So, a piecewise function for g(x) = 2 x 5 + 4 is


g(x) =

2x 6,
{2x
+ 14,

if x < 5
if x 5.

4.7 Exercises (pp. 222224)


Vocabulary and Core Concept Check

if 0 < x 1
if 1 < x 2
if 2 < x 3
if 3 < x 4

1. Sample answer: A piecewise function is a function defined

by two or more equations. Each piece of the function


applies to a different part of its domain. A step function is a
type of a piecewise function, specifically one that is defined
by a constant value over each part of its domain.

Wood Chipper Rental


Total cost (dollars)

if x 5 < 0

g(x) = 2(x + 5) + 4,

+1

if x 5 < 0
if x 5 0
+5+5

y 1 = 2x 4

100,
150,
f (x) =
200,
250,

5)] + 4,
{ 2[(x
2(x 5) + 4,

g(x) = 2[(x 5)] + 4,

y 1 = 2(x) 2(2)
+1

So, the function g(x) = 2 x 5 + 4 represents the


path of the reference beam.

For x 1, use (2, 1) and (3, 3).


31 2
m = = , or 2
32 1
y y1 = m(x x1)

11.

12 a. g(x) = a x h + k

y
300
200
100
0

Number of days

Copyright Big Ideas Learning, LLC


All rights reserved.

Algebra 1
Worked-Out Solutions

209

Chapter 4
2. Sample answer:
y

9. g(x) = 3

g(0) = 3

f(x) =
x 2

The value of g is 3 when x = 0.

4
2

10. g(x) = 3
4

The graph of an absolute function, such as the graph of


f (x) = x 2 shown, is symmetric about the line x = h, or
x= 2 in this example. So, the graph can be split at x = 2,
and a piecewise function can have one equation define the
graph for x < 2 and another equation define the graph for
x 2.
Monitoring Progress and Modeling with Mathematics
3.

g(1) = 3

f (x) = 5x 1

The value of g is 3 when x = 1.


11. g(x) = 2x 5

g(2) = 2(2) 5
g(2) = 4 5
g(2) = 1
The value of g is 1 when x = 2.
12. g(x) = 2x 5

g(5) = 2(5) 5

f (3) = 5(3) 1

g(5) = 10 5

f (3) = 15 1

g(5) = 5

f (3) = 16
The value of f is 16 when x = 3.
4.

f (x) = x + 3

The value of g is 5 when x = 5.


13. Because 2 < 4 5, use g(x) = 65x 20.

g(x) = 65x 20

f (2) = 2 + 3

g(4) = 65(4) 20

f (2) = 1

g(4) = 260 20

The value of f is 1 when x = 2.

g(4) = 240

5. f (x) = x + 3

f (0) = 0 + 3
f (0) = 3

You travel 240 miles in 4 hours on a trip.


14. Because 25 26 < 50, use c(x) = 15.80x + 20.

c(x) = 15.80x + 20

The value of f is 3 when x = 0.

c(26) = 15.80(26) + 20
c(26) = 410.8 + 20

6. f (x) = x + 3

f (5) = 5 + 3

c(26) = 430.8

f (5) = 8

The total cost of ordering 26 shirts is $430.80.

The value of f is 8 when x = 5.


7.

g(x) = x + 4
g(4) = (4) + 4
g(4) = 4 + 4

15.

Open Closed
x

g(4) = 8

2
1

The value of g is 8 when x = 4.


8.

g(x) = x + 4
g(1) = (1) + 4
g(1) = 1 + 4
g(1) = 5
The value of g is 5 when x = 1.

21

1 2 3 4 5 6 x

2
3
4
5
6

The domain is all real numbers.


The range is y 4.

210

Algebra 1
Worked-Out Solutions

Copyright Big Ideas Learning, LLC


All rights reserved.

Chapter 4
16.

19.

Closed Open

Open Closed

Closed Open

10

16

12

8 x

4
4

4 x
8

16

12

The domain is all real numbers.

The domain is all real numbers.

The range is y < 6.

The range is y 2.

17.

8
7
6
5
4
3

20.

Closed Open

Open Closed

2
1
4 3 2

1
21

1 2 3 4 x
2
3

1 2 3 4 5 6 x

The domain is all real numbers.


The range is y 1.

The domain is all real numbers.


The range is y 1 or 0 < y < 3.

18.

10

12

16

20

32

21. Because 5 5, when x = 5, f (x) = x + 8 should be used.

f (x) = x + 8
f (5) = 5 + 8

f (5) = 13

24

The value of f is 13 when x = 5.

16

22. Because x 2 and x > 2, there should be a closed dot at

(2, 4) and an open dot at (2, 1).


4

12 x

6
5
4
3

The domain is all real numbers.


The range is all real numbers.

5 4 3 21

1 2 x

Copyright Big Ideas Learning, LLC


All rights reserved.

Algebra 1
Worked-Out Solutions

211

Chapter 4
23. For x < 0, use (2, 0) and (1, 1).

10
1
10
m = = = , or 1
1 (2) 1 + 2 1

26. For x 2, use (4, 6) and (3, 4).

4 (6) 4 + 6 2
m = = = , or 2
3 (4) 3 + 4 1

y 0 = 1[x (2)]

y y1 = m(x x1)

y = 1(x + 2)

y (4) = 2[x (3)]

y = 1(x) + 1(2)

y + 4 = 2(x + 3)

y=x+2

y + 4 = 2(x) + 2(3)

For x 0, y = 2.

y + 4 = 2x + 6

So, a piecewise function for the graph is


f (x) =

{ x2,+ 2,

if x < 0
.
if x 0

24. For x 0, y = 3.

For x > 0, use (1, 0) and (2, 3).


3 0 3
m = = , or 3
21
1
y y1 = m(x x1)
y 0 = 3(x 1)
y = 3(x) 3(1)
y = 3x + 3
So, a piecewise function for the graph is
f (x) =

{ 3,
3x + 3,

if x 0
.
if x > 0

25. For x < 4, use (2, 2) and (3, 3).

3 (2) 3 + 2 1
m = = = , or 1
32
32
1
y y1 = m(x x1)
y (2) = 1(x 2)

y = 2x + 2
For x > 2, use (0, 1) and (2, 0).
0 (1) 0 + 1 1
m===
20
20 2
y y1 = m(x x1)
1
y 0 = (x 2)
2
1
1
y = (x) (2)
2
2
1
y=x1
2
So, a piecewise function for the graph is
f (x) =

2x + 2, if x 2
.
1
x 1, if x > 2
2

27. For x 2, y = 1.

For 2 < x 0, use (1, 2) and (0, 0).


0 (2) 0 + 2 2
m = = = , or 2
0 (1) 0 + 1 1

y + 2 = 1(x) 1(2)

y y1 = m(x x1)

y + 2 = x + 2

y 0 = 2(x 0)

y = 2(x)
y = 2x

y = x
For x 4, use (5, 4) and (6, 5).

For x > 0, use (2, 1) and (4, 0).

1
5 (4) 5 + 4
m = = = , or 1
65
1
65

0 1 1
m==
42
2

y y1 = m(x x1)
y (4) = 1(x 5)
y + 4 = 1(x 5)
y + 4 = 1(x) 1(5)
y + 4 = x + 5
4

y = x + 1

y y1 = m(x x1)
1
y 0 = (x 4)
2
1
1
y = (x) (4)
2
2
1
y = x + 2
2
So, a piecewise function for the graph is

So, a piecewise function for the graph is

x,
f (x) =
x + 1,

212

if x < 4
.
if x 4

Algebra 1
Worked-Out Solutions

f (x) =

1,
2x,
1
x + 2,
2

if x 2
if 2 < x 0
.
if x > 0

Copyright Big Ideas Learning, LLC


All rights reserved.

Chapter 4
y

32.

28. For x 1, use (3, 1) and (2, 2).

1 2 3 4 5 6 7 8 x

21
1
21
m = = = , or 1
2 (3) 2 + 3 1

y y1 = m(x x1)

y 2 = 1[x (2)]

12

y 2 = 1(x + 2)

16

y 2 = 1(x) + 1(2)

The domain is 1 < x 5.

y2=x+2
+2

The range is 10, 8, 6, and 4.

+2

y=x+4

33.

For 1 < x < 3, use the open dots at (1, 0) and (3, 1).

16

12

1 0 1
1 0
m===
3 (1)
3+1
4

y y1 = m(x x1)

1
y 0 = [x (1)]
4
1
y = (x + 1)
4
1
1
y = (x) (1)
4
4
1
1
y = x
4
4

12

16 x

The domain is 1 < x 12.


The range is 1, 5, 6, and 9.
34.

For x 3, y = 3.
8

So, a piecewise function for the graph is

x
2

x + 4,
if x 1
1
1
f (x) = x , if 1 < x < 3.
4
4
3,
if x 3

The domain is 6 x < 0.


The range is 2, 1, 0, and 1.

29. A step function for the graph is

5,
f (x) = 3,
1,

if 5 x < 3
if 3 x < 1.
if 1 x < 1

4,
3,
30. A step function for the graph is f (x) =
2,
1,
31.

8
7
6
5
4
3

if 0 < x 1
if 1 < x 2
.
if 2 < x 3
if 3 < x 4

2
1
1 2 3 4 5 6 7 8 x

The domain is 0 x < 8.


The range is 3, 4, 5, and 6.

Copyright Big Ideas Learning, LLC


All rights reserved.

Algebra 1
Worked-Out Solutions

213

Chapter 4
Number
of team
members

35.

39. g(x) =

Total cost
(dollors)

0<x5

180

5<x6

210

6<x7

240

7<x8

270

8<x9

300

180,
210,
f (x) = 240,
270,
300,

if 0 < x 5
if 5 < x 6
if 6 < x 7
if 7 < x 8
if 8 < x 9

Total cost (dollars)

g(x) = x 2, if x 2 0

+2+2

+2+2

g(x) = x + 2, if
g(x) =

{ xx +2,2,

x2

if x < 2
.
if x 2

+ 5),
{ (x
x + 5,

if x + 5 < 0
if x + 5 0

g(x) = (x + 5), if x + 5 < 0

g(x) = x + 5, if x + 5 0

250

55

55

200

g(x) = x 5, if

150

So, a piecewise function for y = x + 5 is

100

g(x) =

10

41. g(x) =

12

Number of people on team

Number of
hours

1<x2

2<x3

12

3 < x 24

15

{ xx +5,5,

x < 5 g(x) = x + 5, if

+ 3)],
{ 2[(x
2(x + 3),

g(x) = 2[(x + 3)],

4,
8,
f (x) =
12,
15,

x 5

if x < 5
.
if x 5
if x + 3 < 0
if x + 3 0
if x + 3 <

g(x) = 2(x 3),

Total cost
(dollors)

0<x1

0
3

g(x) = 2(x) 2(3),


if 0 < x 1
if 1 < x 2
if 2 < x 3
if 3 < x 24

g(x) = 2x 6,

if x < 3

g(x) = 2(x + 3),

if x + 3

g(x) = 2(x) + 2(3),

g(x) = 2x + 6,

0
3

x 3

if

So, a piecewise function for y = 2 x + 3 is

Total cost (dollars)

g(x) = x 2, if

300

g(x) =

16
12

42. g(x) =

6,
{ 2x
2x + 6,

g(x) = 4[(x 1)],

g(x) = 4(x + 1),

12

16

20

24

37. A piecewise function for y = x + 1 is

g(x) =

{ xx ++1,1,

if x < 0
.
if x 0

38. A piecewise function for y = x 3 is

g(x) =

3,
{ x
x 3,

if x < 0
.
if x 0

Algebra 1
Worked-Out Solutions

if x < 3
.
if x 3

1)],
{ 4[(x
4(x 1),

Time parked (hours)

214

x<2

So, a piecewise function for y = x 2 is

50

36.

if x 2 < 0
if x 2 0

g(x) = (x 2), if x 2 < 0

40. g(x) =

2),
{ (x
x 2,

if x 1 < 0
if x 1 0
if x 1 <

+1 +1

g(x) = 4(x) + 4(1),


g(x) = 4x + 4,

if

g(x) = 4(x 1),

if x 1

g(x) = 4(x) 4(1),


g(x) = 4x 4,

x<1
+1

if

0
+1

x1

So, a piecewise function for y = 4 x 1 is


g(x) =

+ 4,
{ 4x
4x 4,

if x < 1
.
if x 1

Copyright Big Ideas Learning, LLC


All rights reserved.

Chapter 4
43. g(x) =

8)],
{ 5[(x
5(x 8),

g(x) = 5[(x 8)],

if x 8 < 0
if x 8 0
if x 8 <

g(x) = 5(x + 8),

+8

46. g(x) =

g(x) = 7[(x + 1)] 5,

g(x) = 5(x 8),

if x 8

g(x) = 5(x) 5(8),


g(x) = 5x + 40,

0
+8

So, a piecewise function for y = 5 x 8 is


g(x) =

5x 40,
{5x
+ 40,

3[(x + 6)],
44. g(x) =
3(x + 6),
g(x) = 3[(x + 6)],

if x < 8
.
if x 8

g(x) = 3(x 6),

g(x) = 3(x + 6),

if x + 6

g(x) = 3(x) 3(6),

g(x) = 7x + 7 5,
g(x) = 7x + 2,

x 1

if

12,
{ 7x
7x + 2,

if x < 1
.
if x 1

g(x) = a x 3 + 0

x < 6
6

2 = a 1
2 = a(1)

2=a

x 6

if

So, a piecewise function for y = 3 x + 6 is

45. g(x) =

if x + 1 0

2 = a 4 3 + 0
if

x < 1

47. a. g(x) = a x h + k

g(x) = 3x + 18,

3x + 18,
g(x) =
3x 18,

g(x) = 7(x + 1) 5,

g(x) =

g(x) = 3(x) 3(6),

g(x) = 3x 18,

if

So, a piecewise function for y = 7 x + 1 5 is

if x + 6 < 0
if x + 6 0
if x + 6 <

0
1

g(x) = 7x 12,
g(x) = 7(x) + 7(1) 5,

x8

if

g(x) = 7x 7 5,

x<8
+8

if x + 1 <

g(x) = 7(x) 7(1) 5,

g(x) = 5(x) 5(8),


if

if x + 1 < 0
if x + 1 0

g(x) = 7(x 1) 5,

+8

g(x) = 5x 40,

+ 1)] 5,
{ 7[(x
7(x + 1) 5,

if x < 6
.
if x 6

3)] + 2,
{ [(x
(x 3) + 2

g(x) = [(x 3)] + 2,

if x 3 <
+3

g(x) = x 1,

if

g(x) = (x 3) + 2,

if x 3

g(x) = x + 5

if x 3 < 0
if x 3 0
if x 3 < 0
+3 +3

g(x) = 2(x) + 2(3),


0

+3

g(x) = 2x + 6,

if

g(x) = 2(x 3),

if x 3

g(x) = 2(x) 2(3),

x<3
+3

if

3)],
{ 2[(x
2(x 3),

g(x) = 2(x + 3),

g(x) = x 3 + 2,

g(x) = x + 3 + 2

b. g(x) =

g(x) = 2[(x 3)],

if x 3 < 0
if x 3 0

g(x) = (x + 3) + 2,

So, the function g(x) = 2 x 3 represents the path of the


sunlight that reflects off the water.

g(x) = 2x 6,
0

+3

x3

x<3

+3
if

+3

x3

So, a piecewise function for g(x) = 2 x 3 is


g(x) =

+ 6,
{ 2x
2x 6,

if x < 3
.
if x 3

So, a piecewise function for y = x 3 + 2 is


g(x) =

x 1,
{ x
+ 5,

if x < 3
.
if x 3

Copyright Big Ideas Learning, LLC


All rights reserved.

Algebra 1
Worked-Out Solutions

215

Chapter 4
48. a.

g(x) = a x h + k
g(x) = a x 6 + 4
2 = a 9 6 + 4
2 = a 3 + 4
2 = a(3) + 4
2 = 3a + 4
4

2 = 3a
2 3a
=
3
3
2
= a
3
2
So, the function g(x) = x 6 + 4 represents the path
3
of the golf ball.

2
[(x 6)] + 4, if x 6 < 0
3
b. g(x) =
2
if x 6 0
(x 6) + 4,
3
2
if x 6 < 0
g(x) = (x + 6) + 4,
3
2
2
+6 +6
g(x) = (x) (6) + 4,
3
3
2
g(x) = x 4 + 4,
3
2
if
x<6
g(x) = x,
3
2
if x 6 0
g(x) = (x 6) + 4,
3
2
2
+6 +6
g(x) = (x) (6) + 4,
3
3
2
g(x) = x + 4 + 4,
3
2
if
x6
g(x) = x + 8,
3
2
So, a piecewise function for g(x) = x 6 + 4 is
3
2
if x < 6
x,
.
g(x) = 3
2
x + 8, if x 6
3

49. a. For x 3, use (0, 1) and (2, 2).

21 1
m==
20 2
y y1 = m(x x1)
1
y 1 = (x 0)
2
1
y 1 = x
2
+1

+1

1
y = x + 1
2
1
So, f (x) = x + 1 represents the graph for x 3.
2
1
f (x) = x + 1
2
1
f (10) = (10) + 1
2
= 5 + 1
= 4
So, f (10) = 4.
b. For x > 3, use (4, 2) and (5, 3).

32 1
m = = , or 1
54 1
y y1 = m(x x1)
y 2 = 1(x 4)
y 2 = 1(x) 1(4)
y2=x4
+2

+2

y=x2
So, f (x) = x 2 represents the graph for x > 3.
f (x) = x 2
f (8) = 8 2
=6
So, f (8) = 6.
50. a. When the inequality symbols are changed, the open dot at

(2, 4) becomes a closed dot, and the closed dot at (2, 3)


becomes an open dot. The domain is still all real numbers.
The range of the original equation was y < 4, but when
the open dot at (2, 4) becomes a closed dot, the range
becomes y 4.
b. Because both parts of the graph meet at the point (1, 2),

changing the inequality symbols will not change the


graph. So, the domain and range will also stay the same.

216

Algebra 1
Worked-Out Solutions

Copyright Big Ideas Learning, LLC


All rights reserved.

Chapter 4
51.

x + 2

(4) + 2

3
2

53.

Domain

Range

4 x < 3

(3) + 2

3 x < 2

(2) + 2

2 x < 1

1 x < 0

Open dot:

8
7
6
5
4
3

|x|

0x<1

1x<2

2x<3

3x<4

4
3
2
1

y
4 3 2

1 2 3 4 x
3
4

2
1
4 3 21

The greatest integer function is both a piecewise function


and a step function. It is a piecewise function because each
piece of the function applies to a different part of the
domain. More specifically, it is a step function, because it is
defined by a constant value over each part of its domain,
and therefore the graph consists of a series of horizontal
line segments.

1 2 3 4 x

The domain is all real numbers.


The range is y 0.
52. a. It costs more to make 100 copies than 101 copies.

The closed dot at (100, 10) represents a cost of $10


for 100copies. The point that represents the cost of
101copies is on the next portion of the graph, and it has
ay-value that is less $10.
b. Yes: The portion of the graph that is farthest to the

right has a point on it with a y-value of $40, and the


corresponding x-value is greater than 500. So, with $40,
you can buy more than 500 copies.

Copyright Big Ideas Learning, LLC


All rights reserved.

54.

x 2x 2
1 2(1) 2
2 2(2) 2
3 2(3) 2

y
0
2
4

x y
3 3
4 3
5 3

Because there are closed dots at both (3, 4) and (3, 3), the
same x-value, 3, is paired with two different y-values, 3
and 4. So, it is not a function. If you replace with < or
with >, then one will be a closed dot and the other will
be an open dot so that no vertical line can be drawn through
more than one point on the graph, which means that y will
represent a function.

Algebra 1
Worked-Out Solutions

217

Chapter 4
55. a. x

58.

1
2

y
1(1)
1(2)

y
1
2

2 + 2(3 2)

2 + 2(4 2)

2 + 2(8 2)

14

9 14 + 1(9 8)

15

h(x) = 4x + 3

4 3 2

The transformations are a vertical stretch by a factor of 4 and


then a vertical translation 3 units up.
59.
h(x) = x 8

Depth of snow (inches)

So, a piecewise function that represents the depth of snow is


x,
if 0 x 2
f (x) = 2x 2, if 2 < x 8.
x + 6,
if 8 < x 9

6 5 4 3 2

x
2
3

f(x) = x

60.

0 1 2 3 4 5 6 7 8 9 x

b. no; The point (9, 15) represents the total accumulation of

15 inches in 9 hours. There were 2 inches of accumulation


in the first 2 hours, an additional 12 inches in the next
6hours, and 1 more inch in the last hour for a total of
2 + 12 + 1 = 15 inches.
Maintaining Mathematical Proficiency

{ {

56. A number r is greater than 12 and no more than 13.

t4

t 18

r > 12

r 13

and

An inequality is 12 < r 13.

4
5
6

12

16

or

An inequality is t 4 or t 18.
0

12

16

20

24

6
5
4
3

f(x) = x
h(x) = 12 x + 5

2
1
1 2 3 4 5 6 7 8 x
2

Sample answer: The translations are a vertical shrink by a


factor of 12, followed by a reflection in the y-axis and then a
vertical translation 5 units up.
4.44.7 What Did You Learn? (p. 225)
1. Sample answer: You could use the Internet or an almanac to

search for statistics or other data about your favorite sport or


pastime.

57. A number t is less than equal to 4 or no less than 18.

y
2
1

Sample answer: The translations are a reflection in the y-axis


followed by a vertical translation 8 units down.

Time (hours)

1 2 3 4 x
2
3

{
{

16 12

f(x) = x

1(x),
if 0 x 2
if 2 < x 8
f (x) = 2 + 2(x 2),
14 + 1(x 8), if 8 < x 9

y
16
14
12
10
8
6
4
2
0

2. Sample answer: You first subtract the values of consecutive

terms to find the common difference. Then you repeatedly


add this amount to the last term in order to get the next.
If you write an equation for an in the form
an = a1 + (n 1)d, then you can use this equation to
calculate the value of a term, such as the 50th term, a50,
without knowing the values of all the terms before it.
3. Use the definitions of a piecewise function and a step

function. A piecewise function is broken up into two or more


pieces that are each defined by a separate equation over
the corresponding part of the domain. A step function is a
type of piecewise function, whose pieces are horizontal
segments, each of which are defined by a different
constant value.

218

Algebra 1
Worked-Out Solutions

Copyright Big Ideas Learning, LLC


All rights reserved.

Chapter 4
Chapter 4 Review (pp. 226228)

3 4 1
40
4
0 1 1
Line b: m = =
40
4
40 4
Line c: m = = , or 2
42 2

6. Line a: m = =

1. Let (x1, y1) = (0, 1) and (x2, y2) = (4, 1).

1
y2 y1 1 1 2
m=
= = , or
2
x2 x1
40
4
Because the line crosses the y-axis at (0, 1), the y-intercept
is1.
1
So, the equation is y = x + 1.
2
2. y y1 = m (x x1)

y 7 = 1(x 4)

Lines a and b have slopes of 4 . So, they are parallel. None


of the lines have negative reciprocal slopes, so none are
perpendicular.
7. Line a:

2x 7y = 14

2x 2x 7y = 14 2x

The equation is y 7 = (x 4).

7y = 14 2x

3. Write f (10) = 5 as (10, 5) and f (2) = 3 as (2, 3). Find

14 2x
7
2
y = 2 + x,
7

7y
7

the slope of the line through these points.


3 5 8
m = = , or 1
2 10
8
y y1 = m(x x1)

Line b:

7
y = x 8
2

y 5 = 1(x 10)

Line c:

2x + 7y = 21

y 5 = 1(x) 1(10)

7y = 21 2x
21 2x
7
2
y = 3 x, or
7

7y
7

y=x5
A function is f (x) = x 5.
4. Write f (3) = 4 as (3, 4) and f (5) = 4 as (5, 4). Find

4 (4) 4 + 4 0
m = = = , or 0
53
53
2
y y1 = m(x x1)

8. y y1 = m(x x1)

y+4=0
44

y 5 = 4(x 1)
y 5 = 4(x) 4(1)

y = 4

y 5 = 4x + 4

A function is f (x) = 4.

+5

An equation of the parallel line is y = 4x + 9.

of the line through these points.

5
y 8 = (x 6)
3
5
5
y 8 = (x) (6)
3
3
5
y 8 = x + 10
3
+8
+8
5
y = x + 18
3

+5

y = 4x + 9

5. Write f (6) = 8 as (6, 8) and f (9) = 3 as (9, 3). Find the slope

y y1 = m(x x1)

2
y = x 3
7

The slope of line b is 72 . The slope of line c is 27, which


is thenegative reciprocal of 72 . So, lines b and c are
perpendicular. The slope of line a is 27 , which is not the
sameas either of the other two slopes. So, none of the lines
are parallel.

the slope of the line through these points.

3 8 5
m==
96
3

2
y = x 2
7

2x 2x + 7y = 21 2x

y 5 = x 10
+5
+5

y (4) = 0(x 3)

or

9.

y = mx + b
1
3 = (2) + b
2
3 = 1 + b
1 1
4 = b
Using m = 12 and b = 4, an equation of the
perpendicularline is y = 12 x 4.

10. The roasting time for a 12-pound turkey is 4 hours.

5
A function is f (x) = x + 18.
3
Copyright Big Ideas Learning, LLC
All rights reserved.

Algebra 1
Worked-Out Solutions

219

Chapter 4
11. Sample answer:

b. y = 0.50x 23.5

Use (8, 3.0) and (20, 5.5).

y = 0.50(60) 23.5

5
5.5 3.0 2.5
The slope of the line is m = = = .
20 8
12
24
y y1 = m(x x1)
5
y 3 = (x 8)
24

y = 30 23.5
y = 6.5
A student, who is 60 inches tall, has a shoe size of about
6.5.

5
5
y 3 = (x) (8)
24
24

14. no; Height does not determine shoe size.

5
5
y3=x
24
3
+3
+3

15. d = 10 11 = 1

an = a1 + (n 1)d
an = 11 + (n 1)(1)

4
5
y=x+
3
24

an = 11 + n(1) 1(1)
4

5
An equation of a line that models the data is y =
x + 3 .
24
5
The slope of this line is
,which
means
that
the
roastingtime
24
5
increases by about
hour for each pound the weight ofthe
24
turkey increases. The y-intercept is 43, which has no meaning
in this context because the weight of a turkey cannot be
0 pounds.

12.

an = 11 n + 1
an = n + 12
An equation for the nth term of the arithmetic sequence
is an= n + 12.
an = n + 12
a30 = (30) + 12
a30 = 30 + 12

y-Value
from model

Residual

64

8.5

9 8.5 = 0.5

62

7.5

7 7.5 = 0.5

70

12

11.5

12 11.5 = 0.5

63

88=0

72

13

12.5

13 12.5 = 0.5

68

9.5

10.5

9.5 10.5 = 1

an = 6 + 6n 6

66

9.5

9 9.5 = 0.5

an = 6n

74

13.5

13.5

13.5 13.5 = 0

68

10

10.5

10 10.5 = 0.5

An equation for the nth term of the arithmetic sequence


is an= 6n.

59

6.5

6.5 6 = 0.5

a30 = 18
The 30th term of the arithmetic sequence is 18.
16. d = 12 6 = 6

an = a1 + (n 1)d
an = 6 + (n 1)(6)
an = 6 + n(6) 1(6)

an = 6n
a30 = 6(30)
a30 = 180

residual

The 30th term of the arithmetic sequence is 180.


60

62

64

66

68

70

72

74 x

The points are evenly dispersed about the horizontal axis. So,
the equation y = 0.50x 23.5 is a good fit.
13. a.

y = 0.50x 23.5
9 = 0.50x 23.5
+ 23.5
+ 23.5
32.5 = 0.50x
32.5
0.50

0.50x
0.50

65 = x
A student whose shoe size is 9 is about 65 inches tall.

17. d = 6 (9) = 6 + 9 = 3

an = a1 + (n 1)d
an = 9 + (n 1)(3)
an = 9 + n(3) 1(3)
an = 9 + 3n 3
an = 3n 12
An equation for the nth term of the arithmetic sequence
is an= 3n 12.
an = 3n 12
a30 = 3(30) 12
a30 = 90 12
a30 = 78
The 30th term of the arithmetic sequence is 78.

220

Algebra 1
Worked-Out Solutions

Copyright Big Ideas Learning, LLC


All rights reserved.

Chapter 4
3

18. a. Because 0 0, use y = 2 x + 3.

22. g(x) =

3
y = x + 3
2
3
y = (0) + 3
2
=0+3

g(x) = 4(x + 5),

if x + 5

g(x) =

The value of y is 10 when x = 5.


x
x+6
2 2 + 6
1 1 + 6
0
0+6

y
4
5
6

open dot:

23. g(x) =

x 3x
y
0 3(0)
0
1 3(1) 3
2 3(2) 6

0
5

x 5

if

20,
{ 4x
4x + 20,

if x < 5
.
if x 5

+ 2)] 3,
{ 2[(x
2(x + 2) 3,

if x + 2 < 0
if x + 2 0

g(x) = 2[(x + 2)] 3,

if x + 2 <

g(x) = 2(x 2) 3

0
2

g(x) = 2(x) 2(2) 3

6
5
4
3

g(x) = 2x 4 3

2
1
5 4 3 21

g(x) = 2x 7,

if

x < 2

g(x) = 2(x + 2) 3,

if x + 2

g(x) = 2(x) + 2(2) 3

1 2 x

0
2

g(x) = 2x + 4 3
g(x) = 2x + 1,

The domain is all real numbers.

4x + 2

2x 6

6 4(6) + 2 22

4 2(4) 6 14

5 4(5) + 2 18

3 2(3) 6 12

4 4(4) + 2 14

2 2(2) 6 10

g(x) =

8 x

24

The domain is all red numbers.


The range is all real numbers.
21. A piecewise function for y = x + 15 is

x + 15,
g(x) =
x + 15,

if x < 0
.
if x 0

Copyright Big Ideas Learning, LLC


All rights reserved.

if x < 2
.
if x 2

0<x1

65

1<x2

100

2<x3

135

65,
f (x) = 100,
135,

if 0 < x 1
if 1 < x 2
if 2 < x 3

Popcorn Machine Rental

Total cost (dollars)

16

7,
{ 2x
2x + 1,

24. Number of days Total cost

x 2

if

So, a piecewise function for y = 2 x + 2 3 is

The range is y 6.

x < 5

A piecewise function for y = 4 x + 5 is

= 10

if

g(x) = 4x + 20,

y = 2(5)

0
5

g(x) = 4x 20,
g(x) = 4(x) + 4(5)

y = 2x

g(x) = 4(x) 4(5)

b. Because 5 > 0, use y = 2x.

if x + 5 <

g(x) = 4(x 5)

The value of y is 3 when x = 0.

20.

if x + 5 < 0
if x + 5 0

g(x) = 4[(x + 5)],

=3

19.

+ 5)],
{ 4[(x
4(x + 5),

y
160
140
120
100
80
60
40
20
0

Days

Algebra 1
Worked-Out Solutions

221

Chapter 4
Chapter 4 Test (p. 229)
1.

2x + 4

6.

3 2(3) + 4 2

1
3

x 1

1
3

(0) 1

1 2(1) + 4

(3) 1

Using m = 4 and b = 5, the equation of the perpendicular


line is y = 4x + 5.
7. y y1 = m(x x1)

1 2
6 (3)

1 2
6+3

3
9

1
3

8. m = = = , or

8 x

y y1 = m(x x1)

12

The domain is all real numbers. The range is all real


numbers.

or

y y1 = m(x x1)
1
y (1) = (x 6)
3
1
y + 1 = (x 6)
3

1
y 2 = (x (3))
3
1
y 2 = (x + 3)
3
1

The equation is y 2 = 3(x + 3) or y + 1 = 3(x 6).

9. a1 = 42 and d = 3
2

10

12

The domain is 0 x 12 and the range is 2, 1, 0, 1.


3. y = mx + b

an = 42 + (n 1)(3)
an = 42 + n(3) 1(3)
an = 42 + 3n 3
a. an = 3n + 39

2
The equation is y = x 7.
5
9
3

3 6
30

4. m = = , or 3

y y1 = m(x x1)
y 6 = 3(x 0)
y 6 = 3(x)
y 6 = 3x
+6

an = a1 + (n 1)d

an = 3n + 39

2
y = x + (7)
5

a25 = 3(25) + 39
a25 = 114
So, row 25 has 114 seats.
b. an = 3n + 39

90 = 3n + 39
39

39

51 = 3n
51
3

3n
3

+6

y = 3x + 6
The equation is y = 3x + 6.
5.

+4

5=b

The equation is y 2 = 10(x 6).

2
1

+4

y 2 = 10(x 6)

2.

1 = 4 + b

1
3

1 = 4(1) + b

1
1
open dot: 1
(1) 1 1
3
3

2 2(2) + 4

y = mx + b

17 = n
So, row 17 has 90 seats.

y y1 = m(x x1)
y (8) = 3(x (2))
y + 8 = 3(x + 2)
y + 8 = 3(x) + 3(2)
y + 8 = 3x + 6
8

y = 3x 2
The equation of the parallel line is y = 3x 2.

222

Algebra 1
Worked-Out Solutions

Copyright Big Ideas Learning, LLC


All rights reserved.

Chapter 4
d. There may be a casual relationship in the data but the

Neighborhood Festival Attendance

Yearly attendance

10. a.

correlation may be caused by other factors, such as the


quality of the attractions each year.

y
1600

e.

1200

2000 = 0.19x + 309

800

309

309

1691 = 0.19x

400
0

y = 0.19x + 309

1000 2000 3000 4000

8900 = x

Advertising (dollars)

The scatter plot shows a positive correlation.


b. Sample answer: A line through (1000, 500) and

(2000,700) models the data.

700 500
200
1
The slope of the line is m = = = .
2000 1000 1000 5

12. Line 1:

y c = ax

y c + c = ax + c
y = ax + c
Line 2:

y y1 = m(x x1)

ay = x b
x b
a
b
1
y = x
a
a

ay
a

1
y 500 = (x 1000)
5
1
1
y 500 = (x) (1000)
5
5
1
y 500 = x 200
5
+ 500

In order to get 2000 people to attend the festival, $8900


should be spent on advertising.

Line 3: ax + y = d
ax ax + y = d ax

+ 500

y = d ax,

1
y = x + 300
5

y = ax + d
1

An equation of the line of fit is y = 15x + 300.


1

c. Sample answer: The slope of the line is 5 . This means that

the yearly attendance at the festival increases by about


1 person for every $5 spent on advertising. The y-intercept
is 300, meaning that if no money is spent on advertising,
about 300 people would attend the festival.

11. a. After entering the data from the table into two lists using

a graphing calculator, the linear regression feature yields


the equation y = 0.19x + 309.
b. The correlation coefficient is about 0.943. This means that

there is a strong positive correlation between the amount


spent on advertising and the yearly attendance of the
festival. So, the equation y = 0.19x + 309 closely models
the data.
c. Sample answer: Because the data show a strong positive

correlation and the line of best fit closely models the data,
you would expect the scatter plot of the residuals to show
a relatively even distribution of residuals on both sides of
the x-axis.

Copyright Big Ideas Learning, LLC


All rights reserved.

or

The slope of line 1 is a. The slope of line 2 is a, which


is the negative reciprocal of a. So, lines 1 and 2 are
perpendicular. The slope of line 3 is a, which is the
1
opposite of a and the reciprocal of a , but neither the same
nor the negative reciprocal of either. So, line 3 is neither
parallel nor perpendicular to the other two lines.
13. Sample answer:

1, if x 3
f (x) = x,
if 3 < x 1
a,
if x > 1
Chapter 4 Standards Assessment (pp. 230231)
1. C; d = 21 24 = 3

an = a1 + (n 1)d
an = 24 + (n 1)(3)
an = 24 + n(3) 1(3)
an = 24 3n + 3
an = 27 3n

Algebra 1
Worked-Out Solutions

223

Chapter 4
2.

3x + 6 8 + 2x
2x

5x 5 7x 9

2x

5x

x+68

4. g(x) = f (x + 2) is of the form y = f (x h), where h = 2.

So, the graph of g is a horizontal translation 2 units left of


the graph of f, which is represented by the green line.

5x
5 2x 9

6 6

+9

x2

+9

h(x) = f (3x) is of the form y = f (ax), where a = 3. So, the


graph of h is a horizontal shrink of the graph of f by a factor
of 1 3 = 13, which is represented by the red line.

4 2x
4
2

2x
2

k(x) = f (x) + 4 is of the form y = f (x) + k, where k = 4. So,


the graph of h is a vertical translation 4 units up of the graph
of f, which is represented by the purple line.

2x
12 3x 18
12

q(x) = f (x) is of the form y = f (x). So, the graph of q is a


reflection in the x-axis of the graph of f, which is represented
by the blue line.

12
3x 6
3x 6

3
3
x 2

5.

y = mx + b

3
2 x 3 x
2
3
3
+ x
+x
2
2
1
2 3 + x
2
+3

1
1 = (6) + b
2

2 3 1
3
1 + = + =
2
2 2 2

+3

1
2 1 2 x
2

Situation Correlation
a.

No

Causation
Yes

No

{ 2x2x + 3,3,

c.

if x < 0
.
if x 0

The left portion of the graph has a y-intercept of positive 3,


and an open dot where x = 0. The right portion of the graph
has a y-intercept of negative 3 and a closed dot where x = 0.
7. Sample Answer:

My function:

b.
d.

2 = b

y=

Three of the inequalities are equivalent because they have


the same solution, x 2. They are 3x + 6 8 + 2x,
5x 5 7x 9, and 2 32x 3 x.

Yes

33

6. A piecewise function that represents the graph is

2x

3.

1=3+b

Using m = 12 and b = 2, the equation of the line is


y = 12 x 2.

1
1 x
2

1
3 1 4
m = = , or
2 6 8
2

x 3 2 1

y 4

Friends relation:

a. Increasing the price of a pair of pants will cause fewer

people to buy them.


b. Increasing the number of cell phones in a city will not

cause the number of taxis in the city to increase.


c. A persons IQ, or intelligence level, is not related to the

persons athletic ability.


d. Spending more time studying affects the score earned.

x 3 2 3 2 3
y

8. Let x be how much your total bill is before the discount.

0.80x = x 5
x x
0.2x = 5
0.2x
0.2

(0.80 1.00 = 0.2)

5
0.2

x = 25
Both coupons will save you the same amount of money on a
total bill of $25.

224

Algebra 1
Worked-Out Solutions

Copyright Big Ideas Learning, LLC


All rights reserved.

Chapter 4
9. a.

y-Value
from model

Residual

54

40

112

40 112 = 72

60

120

130

120 130 = 10

68

180

154

180 154 = 26

72

260

166

260 166 = 94

78

280

184

280 184 = 96

84

260

202

260 202 = 58

92

220

226

220 226 = 6

98

180

244

180 244 = 64

The points that appear on a scatter plot of the residuals


are (92, 6), (78, 96), (60, 10), (84, 58), (98, 64),
(72, 94), (54, 72), and (68, 26).
b. no; The residual points form a -shaped pattern,

whichsuggests the data are not linear. So, the equation


y= 3x 50 does not model the data well.

Copyright Big Ideas Learning, LLC


All rights reserved.

Algebra 1
Worked-Out Solutions

225

Potrebbero piacerti anche